Ortho Flashcards
Vitamin C has been shown to decrease the likelihood of which of the following complications following surgery on the foot and ankle in non-diabetic patients?
Nonunion Complex Regional Pain Syndrome, type II Malunion Complex Regional Pain Syndrome, type I Wound infection
Vitamin C has been shown to decrease the likelihood of developing complex regional pain syndrome (CRPS), type 1, when given post-operatively to patients undergoing foot and ankle and wrist surgery.
CRPS is a frequent post-operative complication, with rates varying from 10-37%. Type I CRPS does not have an identifiable nerve lesion, while type II has an identifiable nerve lesion. Multiple studies have shown that vitamin C decreases rates of CRPS following distal radius fractures, and more recently, the same has been shown following foot and ankle surgery. While the exact mechanism of CRPS is unknown, vitamin C has been shown to reduce lipid peroxidation, scavenge hydroxyl radicals, protect the capillary endothelium, and inhibit vascular permeability. All of these characteristics of vitamin C may play a role in modulating the pain pathway.
Zollinger et al. perform a double-blind, prospective, multicenter trial where 416 patients with 427 wrist fractures were randomly allocated to treatment with placebo or treatment with 200, 500, or 1500 mg of vitamin C daily for fifty days. The prevalence of complex regional pain syndrome was 2.4% in the vitamin C group and 10.1% in the placebo group.
Besse et al. compare two groups of patients undergoing surgery on the foot and ankle to determine the effect of vitamin C on the development of CRPS, type I. CRPS type I occurred in 18 cases (9.6%) in the group not given vitamin C, and 4 cases (1.7%) in the group given vitamin C.
Increased swelling is a common physical exam finding in patients afflicted with the disease.
Incorrect Answers:
Answers 1, 2, 3, 5: Vitamin C has not been shown to decrease the incidence of these conditions.
A 47-year-old male presents with back pain of 2 weeks duration. He denies night sweats, fevers, or weight loss. He localizes his symptoms to a dermatomal distribution along the rib cage on the right. On physical exam he has mild paraspinal tenderness, normal patellar reflexes, normal muscle strength in his lower extremities, and a normal gait exam. An MRI is shown in Figure A and B. What is the most appropriate first step in management?
A repeat MRI with gadolinium
CT of chest, abdomen, and pelvis followed by a CT guided biopsy of the spinal lesion
Physical therapy and NSAIDs
Surgical decompression using a midline posterior approach
Surgical decompression using a transthoracic approach
Physical therapy and NSAIDs
The clinical presentation and imaging studies are consistent with a thoracic disc herniation. Although less common than lumbar disc herniation, thoracic disc herniations are a recognized cause of back pain. For patients with prolonged symptoms, Blumenkopft and Maiman contend that MRI is the study of choice in evaluation of disc herniations. The downside of MRI is that it can have high false positive rates. In a retrospective study, Wood et. al showed that in asymptomatic individuals 73% had thoracic disc abnormalities and 37% showed frank herniations, 29% of these had cord compression. 75% of herniations occur between T8 and T12. Because the MRI in this question is highly characteristic of a thoracic disc herniation, the suspicion for a malignant or infectious process is low and an MRI with gadolinium, biopsy, and cancer staging are not indicated. The majority of these patient improve with nonoperative management including physical therapy. Because this patient has no neurologic deficits, surgery would not be indicated.
Which of the following is least likely to predict future amputation in diabetic patients?
Diabetic foot ulceration Loss of sensation with 5.07 Semmes-Weinstein monofillament testing Infection Hemoglobin A1c level of 10.7 Ankle-brachial index of 1.07
Ankle-brachial index of 1.07
The references state that diabetic patients who underwent amputations also had these concurrent variables: ulceration 84%, neuropathy 64%, infection 59%, gangrene 55%, and ischemia 46%.
Other risk factors factors included elevated hemoglobin A1c level, decreased oxygen tension levels, and a decreased ankle-brachial index.
Ulceration in diabetic foot due to lack of protective sensation
incidence:
- approximately 12% of diabetics have foot ulcers
most common medical complication causing diabetics to get medical treatment
- foot ulcers are responsible for ~85% of lower extremity amputations
Risk factors
- factors associated with decreased healing potential: uncontrolled hyperglycemi, inability to offload the affected area, poor circulation, infection, poor nutrition
- factors associated with increased healing potential: serum albumin > 3.0 g/dL, total lymphocyte count > 1,500/mm3
Pathophysiology:
Neuropathy has largest effect on diabetic foot pathology:
- Sensory dysfunction leads to lack of protective sensation and is primary risk factor for ulcer development
- Autonomic dysfunction leads to drying of skin due to lack of normal glandular function
- Net effect is increased mechanical and axial stress on skin that is more prone to injury due to drying
Angiopathy - lesser effect than neuropathy, >60% of diabetic ulcers have decreased blood flow due to peripheral vascular disease
Associated conditions
infection / osteomyelitis: high rates of associated osteomyelitis if bone is able to be probed, or is exposed at the base of the ulcer, 67% of ulcers that probe to bone have osteomyelitis. Deep cultures and bacterial biopsies help guide management
Organisms: usually polymicrobial
- gram-positive: most common pathogens are aerobic gram positive cocci (s. aureus)
- gram-negative: increased gram-negative organisms are found in chronic wounds and wounds recently treated with antibiotics
anaerobes: obligate anaerobic pathogens with ischemia or gangrene
Prognosis
diabetic foot ulceration is considered the most likely predictor of eventual lower extremity amputation in patients with diabetes mellitus
Studies
Transcutaneous oxygen pressures (TcpO2)
considered Gold Standard to assess wound healing potential, > 30 mm Hg (or 40mmHg depending on review source cited) is a good sign of healing potential
ABI’s and ischemic index: calcification in the arteries can result in inaccurate doppler flow readings
calcifications falsely elevate the ABI’s due to decreased compliance of the calcified vessels. index of > 0.45 and toe pressure >45mm Hg are needed to heal amputation and >60mm Hg to heal an ulcer
Imaging
Radiographs: AP, lateral, and oblique of foot and ankle
MRI: best for differentiating abscess from soft tissue swelling. Difficult to differentiate infection from Charcot arthropathy on MRI
Bone scan: obtain with technetium Tc99m, gallium (Ga)67, or indium (In) 111
Useful to differentiate between soft tissue infection, osteomyelitis and Charcot arthropathy
A 7-year-old boy sustains a ring finger injury after falling from his bike. The fingernail has been torn transversely beneath the eponychium and the surgeon has removed the nail as shown in Figure A. Radiographs are shown in Figure B. (distal phalanx fracture) What is the next best step in management?
Irrigation and debridement with alumafoam placement and immobilization
Irrigation and debridement followed by percutaneous pinning and immobilization
Irrigation and debridement followed by reduction, nail bed repair and immobilization
Betadine soaks at home three times daily with intermittent alumafoam splint placement and immobilization
Alumafoam splint placement and immobilization
Irrigation and debridement followed by reduction, nail bed repair and immobilization
The clinical presentation is consistent with a physeal separation and a nail bed injury. This is also called a Seymour fracture which is a juxta-epiphyseal fracture of the distal phalanx. Treatment of a nail bed avulsion and physeal separation is irrigation and debridement, physeal reduction, nail bed repair and immobilization. The primary goals are to achieve a stable, viable nail and good cosmetic results.
Inglefield at al retrospectively reviewed 19 children with 22 nail bed injuries. Early operative repair led to good to excellent results in 91% of patients. They concluded that repair of the nail bed at the time of injury is superior to secondary correction.
Fassler reviewed fingertip injuries, providing recommendations for treatment based on degree of soft tissue loss, bone exposure, feasibility for flap coverage and the presence or absence of mitigating systemic conditions. He also concluded that the outcome of nail bed injuries is dependent on the severity of injury to the germinal matrix.
Anatomy
- perionychium: nail, nailbed, surrounding skin
- paronychium: lateral nail folds
- hyponychium: skin distal distal and palmar to the nail
- eponychium: dorsal nail fold, proximal to nail fold
- lunula: white part of the proximal nail
- matrix:
sterile: soft tissue deep to nail, distal to lunula, adheres to nail
germinal : soft tissue deep to nail, proximal to sterile matrix, responsible for most of nail development
Insertion of extensor tendon is approximately 1.2 to 1.4 mm proximal to germinal matrix
A 40 year-old competitive weightlifter felt a painful pop in his elbow while performing a bench press. His lateral radiograph shows ‘flake’ sign. An MRI is likely to show which of the following?
Distal biceps tendon rupture Brachialis muscle rupture Lateral ulnar collateral ligament tear Medial ulnar collateral ligament tear Triceps tendon tear
The lateral radiograph shows a “flake sign” which is common in patients with a triceps tendon avulsion, as suggested by this clinical scenario. Triceps tendon tears occur most commonly in males age 30-50, and those involved in competitive weightlifting and football. There is an established link with anabolic steroid use. Other risk factors include renal disease, corticosteroids, history of tendon injection, and fluroquinolone antibiotics. Surgical repair is indicated in active persons with complete tears and for incomplete tears with concomitant loss of strength.
Yeh et al note the importance of obtaining radiographs and that the flake sign is pathognomonic of a triceps tendon avulsion. They note that an MRI is often used to confirm the diagnosis, classify the injury, and guide management.
van Riet et al reported a case series of triceps tendon ruptures. Making the correct diagnosis is important because primary repair is possible within 3 weeks and affords better results than late reconstruction. A high clinical suspicion and use of imaging is critical, as 10 of their 23 ruptures were missed on initial presentation.
Risk factors
- systemic illness (hyperparathyroidism, renal osteodystrophy, OI, RA, type I DM)
- anabolic steroid use
- local steroid injection
- fluoroquinolone use
- chronic olecranon bursitis
- previous triceps surgery
- Marfan syndrome
Triceps brachii = pennate muscle comprised of 3 heads
- lateral: originates from the posterior humerus between the insertion of the teres minor and the superior aspect of spiral groove, the lateral border of humerus, and the lateral intermuscular septum
- long: originates from the infraglenoid tuberosity
- medial: originates from the posterior humerus distal to spiral groove, the medial humerus, and the medial intermuscular septum
Insertion occurs over a wide area/footprint located 12mm distal to the tip of the olecranon, width ranges from 1.9-4.2cm.
Consists of:
- triceps tendon proper: confluence of tendon from all three heads. inserts on the olecranon
- lateral triceps expansion: medial aspect inserts on the posterior crest of the ulna, adjacent to the medial head;
lateral aspect inserts on the fascia of the extensor carpi ulnaris muscle and the deep fascia of the anconeus muscle; distal aspect inserts on the antebrachial fascia
Only muscle in the posterior compartment of the arm
innervated by radial nerve (C6-C8)
A patient with a severe nickel allergy and degenerative joint disease of the hip would be best served by which of the following prosthetic options?
Cemented titanium stem, ceramic (alumina) head, and press-fit titanium cup
Cemented cobalt-chrome stem, ceramic (alumina) head, and press-fit cobalt-chrome cup
Press-fit titanium stem, cobalt-chrome head, and press-fit titanium cup
Press-fit titanium stem, titanium head, and press-fit titanium cup
Press-fit titanium stem, ceramic (alumina) head, and cementless titanium cup
Press-fit titanium stem, ceramic (alumina) head, and cementless titanium cup
Nickel is present in cobalt-chrome and stainless steel alloys used in orthopaedic surgery; therefore, these materials are not well suited for nickel-sensitive patients. Nickel is not present in titanium alloys or in ceramic components. Titanium is therefore the material of choice for the femoral and acetabular component. Titanium is a poor option for the femoral head due to its susceptibility to abrasive wear. Titanium is also poorly suited to cemented applications in hip arthroplasty because it is less stiff than cobalt-chrome (and stainless steel), and therefore transmits greater stresses to the cement column. Titanium’s poor abrasion resistance can also leave the component susceptible to increased abrasive wear in the event of loosening and micromotion. Therefore, of the options available, a cementless titanium stem and socket with a ceramic head is the best choice. In nickel-sensitive total knee arthroplasty patients, cemented oxidized zirconium femoral components can be used in place of cobalt-chrome, and titanium tibial components (press-fit or cemented) have been used with success.
A 32-year-old carpenter complains of progressively worsening wrist pain for the last 2 months. He denies any recent history of trauma to the wrist or hand. An MRI is obtained and a representative image is provided in Figure A (avascular lunate). Which of the following surgical interventions is thought to be effective for this condition by inciting a local vascular healing response?
Wrist fusion Ulnar shortening osteotomy Distal radius core decompression Proximal row carpectomy Scapholunate ligament reconstruction
This clinical scenario and imaging studies are consistent with Kienbock’s disease, avascular necrosis of the lunate, in the pre-collapse stage. Core decompression of the distal radius is an accepted treatment for Kienbock’s disease. The procedure creates a local vascular healing response facilitating vascular recovery prior to collapse and degeneration of the lunate. Other acceptable interventions include revascularization with a pedicled graft and joint leveling procedures such as a radial shortening osteotomy. The radial shortening osteotomy is ideal for patients with negative ulnar variance who experience greater loads through the radiolunate fossa.
Sherman et al performed a cadaveric study demonstrating minimal change in the distribution of force between the radiocarpal fossa and ulnocarpal fossa following core decompression of the distal radius.
Illarramendi et al reviewed 22 cases of Kienbock’s treated with radial and ulnar metaphyseal core decompression. No surgical complications occurred, and 20 of 22 reported satisfactory clinical outcomes while one patient developed intercarpal arthritis.
Incorrect Answers:
- Proximal row carpectomy and wrist fusion would be options for the collapsed and degenerative lunate.
- Ulnar shortening osteotomy and scapholunate ligament reconstruction are incorrect as they do not address the pathology of Kienbock’s.
- Proximal row carpectomy and wrist fusion would be options for the collapsed and degenerative lunate.
- Ulnar shortening osteotomy and scapholunate ligament reconstruction are incorrect as they do not address the pathology of Kienbock’s
Which of the following tests is required for a standard work-up of Ewing’s sarcoma that is not routinely obtained for staging of osteosarcoma?
MRI CT scan Bone scan Protein electrophoresis Bone marrow biopsy
Bone marrow biopsy is a routine part of the staging workup for Ewing’s sarcoma, and is not routinely obtained for staging of osteosarcoma.
Ewing’s is a small round cell (blue cell) tumor that occurs most commonly in children and young adults. Clinical presentations of these tumors frequently mimic infection with low grade fever, elevated white counts and high markers of inflammation. The radiographic appearance will show a large lytic lesion in the metaphysis or diaphysis. Reactive periosteum may be lifted off the bone in multiple layers, termed “onion skinning” which is characteristic but uncommon. Bone marrow biopsy is done because Ewing’s sarcoma can metastasize via the marrow.
Carvajal et al report the Ewing’s sarcoma family of tumors (EFT) includes ES of bone (ESB), extraosseous ES (EES), peripheral primitive neuroectodermal tumor of bone (pPNET), and malignant small-cell tumor of the thoracopulmonary region, or Askin’s tumor, all of which are now known to be neoplasms of neuroectodermal origin.
Treatment
•Nonoperative: chemotherapy + radiation therapy
◾indications
◾non-resectable tumors (eg. large spinal tumors)
◾sites where functional deficit is unacceptable
◾trend is towards surgical resection / away from RT because of morbidity associated with radiation and risk of secondary malignancies
•Operative
◦chemotherapy + limb salvage resection ± adjuvant radiation ◾indications: standard of care in most patients where primary tumor can be completely removed (expendable and surgically reconstructible sites)
Chemotherapy ◾vincristine, doxorubicin, cyclophosphamide, and dactinomycin
◾preoperative chemotherapy (neoadjuvant) given for 8-12 weeks followed by surgical resection and maintenance (adjuvant) chemotherapy for 6-12 months
Adjuvant radiation ◾not necessary if margins are adequate and there is good response to chemotherapy
◾indications ◾positive post-resection surgical margins
◾patients who present with widely metastatic disease
◾where chemotherapeutic response has been poor
A 45-year-old male trauma patient presents with multiple extremity injuries including the foot injury shown in Figure A (base of 2nd mt intra-artic#). The foot fracture is treated surgically, and heals without any initial complications. At a minimum of 12 months, this patient will be expected to have which of the following scores compared to a matched polytrauma patient without a foot injury?
Lower mean Short Form 36 (SF-36) score
Higher mean score on the AAOS lower limb and foot and ankle outcomes questionnaire
Equivalent score on the Western Ontario and McMaster Universities Osteoarthritis Index (WOMAC)
Lower Constant score
Higher score on the Western Ontario and McMaster Universities Osteoarthritis Index (WOMAC)
Lower mean Short Form 36 (SF-36) score
Turchin et al assessed the outcome of two groups of matched polytrauma patients, with the only difference being the presence of a foot injury in Group 1. They used three outcome tools, SF-36, WOMAC, and Modified Boston Children’s Hospital Grading System to evaluate the two groups at a minimum of 2 years from injury. The foot injury group, including all types of foot fractures, had a poor outcome when using any of these measures. Turchin concludes that “Foot injuries cause significant disability to multiply injured patients. More attention should be given to these injuries, and more aggressive management should be considered to improve the outcome of this group of multiply injured patients.”
Tran et al compared polytrauma patients with foot injuries to 14 polytrauma patients without foot injury at a minimum of 12 months from injury. The AAOS lower limb and foot and ankle outcomes data collection questionnaire showed significantly lower scores in the foot injury group. The Constant score is used for evaluation of shoulder conditions
A 45-year old male is involved in a motor vehicle accident and presents to the emergency room with complaints of neck pain. Physical exam shows he is an ASIA E. An open-mouth cervical radiograph is shown in Figure A. A sagittal CT scan is shown in Figure B. A CT axial angiogram is shown in Figure C (odontoid fracture with an aberrant vertebral artery). Which of the following treatment options is contraindicated in this patient.
Anterior screw osteosynthesis with single cannulated screw
Halo immobilization
Anterior screw osteosynthesis with two cannulated screws
C1-C2 transarticular screws
Posterior C1-C2 wiring with autograft
C1-C2 transarticular screws
The clinical presentation is consistent with an odontoid fracture with an aberrant vertebral artery on the left. C1-C2 transarticular screws are an absolute contraindication in this scenario.
The vertebral artery is an important consideration when performing posterior cervical spine surgery. Injury to this artery can lead to stroke and death. Normally the vertebral artery travels superiorly in the transverse foramen of C6 to C2. At C2 the artery deviates laterally to the pass through the transverse foramen of C1 and then wraps medially on the superior surface of the posterior arch of C1 before ascending into the foramen magnum. Anomalous variants of the vertebral arery may be present in up to 30% of individuals, and may be intraosseous or extraosseous. Intraosseous variants may be injured during posterior cervical stabilization techniques. In patients with an aberrant vertebral artery, C1-C2 transarticular screws are contraindicated due to the risk of injury to the aberrant vertebral artery.
Patel et al. performed a literature review to investigate the optimal surgical treatment for unstable type II odontoid fractures in skeletally mature individuals. They found there is no moderate or high quality literature on the surgical management of acute type II odontoid fractures, and that there is no comparative data to objectively compare an anterior vs. posterior approach. They report that in equivocally indicated instances, anterior or posterior treatment can both be safely used with good outcome.
Wright et al. performed a retrospective study to look at the incidence of vertebral arery injury and subsequent neurological deficit in patients where C1-C2 trans-articular screws are placed. They found of 2492 C1-2 transarticular screws in 1318 patients, thirty-one patients (2.4%) had known vertebral artery injuries and an additional 23 patients (1.7%) were suspected of having injuries. Of the patients with known or suspected vertebral artery injuries, 2 (3.7%) of the 54 patients exhibited subsequent neurological deficits and one (1.9%) died from a bilateral VA injury.
Figure A is an open-mouth odontoid radiograph that shows a type 2 odontoid fracture. Figure B is a sagittal CT that shows a Type 2 odontoid fracture. Figure C is a CT angiogram that shows an anomalous vertebral artery on the left, with the absence of a normal vertebral foramen. Illustration A shows the anatomy of a normal vertebral artery and its relative position to a C1-C2 transarticular screw. Illustration B shows an anomalous extra-osseous vertebral artery. Illustration C shows a CT scan of an anomalous intra-osseous vertebral artery.
Anderson and D’Alonzo Classification
Type I Oblique avulsion fx of tip of odontoid. Due to avulsion of alar ligament. Although rare, atlantooccipital instability should be ruled out with flexion and extension films.
Type II Fx through waist (high nonunion rate due to interruption of blood supply).
Type III Fx extends into cancellous body of C2 and involves a variable portion of the C1-C2 joint.
Axis Osteology ◦axis has odontoid process (dens) and body
◦embryology ◾develops from five ossification centers
◾subdental (basilar) synchondrosis is an initial cartilaginous junction between the dens and vertebral body that does not fuse until ~6 years of age
◾the secondary ossification center appears at ~ age 3 and fuses to the dens at ~ age 12
•Axis Kinematics ◦CI-C2 (atlantoaxial) articulation ◾is a diarthrodial joint that provides ◾50 (of 100) degrees of cervical rotation
◾10 (of 110) degrees of flexion/extension
◾0 (of 68) degrees of lateral bend
◦C2-3 joint ◾participates in subaxial (C2-C7) cervical motion which provides ◾50 (of 100) degrees of rotation
◾50 (of 110) degrees of flexion/extension
◾60 (of 68) degrees of lateral bend
•Occipital-C1-C2 ligamentous stability ◦provided by the odontoid process and its supporting ligaments ◾transverse ligament ◾limits anterior translation of the atlas
◾apical ligaments ◾ limit rotation of the upper cervical spine
◾alar ligaments
◾limit rotation of the upper cervical spine
•Blood Supply ◦a vascular watershed exists between the apex and the base of the odontoid ◾apex is supplied by branches of internal carotid artery
◾base is supplied from branches of vertebral artery
◾the limited blood supply in this watershed area is thought to affect healing of type II odontoid fractures.
A 78-year-old male falls at home four months following a right total hip arthroplasty. Right leg deformity, pain, and inability to bear weight are present on physical exam. An injury radiograph is provided in Figure A (stable stem, distal periprosthetic#), while radiographs taken immediately following the initial total hip arthroplasty are provided in Figures B and C. The patient denies any prodromal groin pain prior to his fall. Which of the following is the best treatment option?
Traction for 6 weeks followed by slow return to weight bearing
Open reduction and internal fixation
Revision to a long, cementless femoral stem
Revision to a long, cementless stem with strut allograft
Revision to a long, cemented stem
Open reduction and internal fixation
The clinical presentation and radiograph are consistent with a Vancouver B1 periprosthetic femur fracture. The stem appears stable within the femur, and there is no evidence of subsidence with comparison to the initial post-THA radiographs. This fracture pattern is best treated with internal fixation. Illustrations A and B are radiographs of this patient following fixation. Illustrations C and D show bone healing at 2 years following the fracture.
Duwelius et al report on 33 periprosthetic femur fractures. All fractures that demonstrated a stable stem at the time of surgery were treated with internal fixation, while those that were unstable were treated with a long, cementless revision femoral stem. At 2.5 years complications were minimal and the patients had regained their pre-fracture level of function.
The review article by Kelley outlines the evaluation, classification, and treatment of periprosthetic femur fractures reinforcing the importance of stem stability within the femur. Periprosthetic fractures around a hemiarthroplasty should be treated with the same algorithm. However, if the patient had antecedent groin pain, then conversion to a total hip arthroplasty should be considered to prevent continued groin pain.
•Fractures around a total hip prosthesis increasing in incidence as a result of increased arthroplasty procedures and high-demands of elderly patients •Classification ◦intraoperative fractures ◾femur ◾acetabulum ◦postoperative fractures ◾femur ◾acetabulum
Intraoperative Acetabular Fractures
Risk factors: underreaming >2mm, elliptical modular cups, osteoporosis, cementless acetabular components, dysplasia, radiation
- Evaluation: must determine stability of implant
- Treatment ◦observation alone if evaluated intraoperatively and found to be stable - consider protected weight-bearing for 8-12 weeks
◦acetabular revision with screws vs. ORIF - if evaluated intraoperatively and found to be unstable
◾technique ◾addition of acetabular screws
◾may consider upgrading to “jumbo” cup
◾ORIF of acetabular fracture with revision of acetabular component ◾if posterior column is compromised, ORIF + revision is most stable construct
◾may add bone graft from reamings if patient has poor bone stock
◾postoperative care ◾consider protected weight-bearing for 8-12 weeks
Intraoperative Femur Fractures
Mechanism
◾proximal fractures: usually occur with bone preparation (ie aggressive rasping) and prosthetic insertion. May occur during implant insertion from dimension mismatch
◾middle-region fractures: usually occur when excessive force is used during surgical exposure or bone preparation
◾distal fractures: usually occur when tip of a straight-stem prosthesis impacting at femoral bow
◦Risk factors: impaction bone grafting,female gender, technical errors, cementless implants, osteoporosis, revision, minimally invasive techniques (controversial)
•Presentation: change in resistance while inserting stem should raise suspicion for fracture
•Classification ◦Vancouver classification (intraoperative) ◾Considerations: location, pattern, stability of fracture
Types:
◾A - proximal metaphysis
◾B - diaphyseal
◾C - distal to stem tip (not amenable to insertion of longest revision stem)
Subtypes
◾1 - cortical perforation
◾2 - nondisplaced crack
◾3 - displaced unstable fracture pattern
•Treatment
◦stem removal, cabling, and reinsertion if intraoperative longitudinal calcar split
◦trochanteric fixation with wires, cables, or claw-plate ◾indications: intraoperative, proximal femur fractures
◦removal of implant, insertion of longer stem prosthesis ◾indications: complete (two-part) fractures of middle region ◾technique ◾distal tip of stem must bypass distal extent of fracture by 2 cortical diameters
◾may use cortical allograft struts for added stability
◦removal of implant, internal fixation with plate, reinsertion of prosthesis ◾indications: distal fractures that cannot be bypassed with a long-stemmed prosthesis
Postoperative Femur fracture
•Incidence 0.1-3% for primary cementless total hip arthroplasties
◦etiology
Early postoperative fractures
◾cementless prosthesis tend to fracture in the first six months - likely caused by stress risers during reaming and broaching
◾wedge-fit tapered designs cause proximal fractures
◾cylindrical fully porous-coated stems tend to cause a distal split in the femoral shaft
Late postoperative fractures
◾cemented prosthesis tend to fracture later (5 years out)
◾tend to fracture around the tip of the prosthesis or distal to it
Risk factors: ◾poor bone quality ◾cementless prostheses ◾compromised bone stock ◾revision procedures
•Classification ◦Vancouver classification (postoperative) ◾considerations ◾stability of prosthesis
◾location of fracture
◾quality of surrounding bone
◾pros: simple, validated
◾cons: often difficult to differentiate between B1 and B2 fractures based on radiographs alone
A: Fracture in trochanteric region. Commonly associated with osteolysis. AG (greater trochanter) fractures caused by retraction, broaching, actual implant insertion, previous hip screws.
Often requires treatment that addresses the osteolysis.
AG fractures with <2cm displacement, treat nonoperatively with partial WB and allow fibrous union.
AG fractures >2cm needs ORIF (loss of abductor function leads to instability) with trochanteric claw/cables.
B1 Fracture around stem or just below it, with a well fixed stem ORIF using cerclage cables and locking plates
B2 Fracture around stem or just below it, with a loose stem but good proximal bone stock Revision of the femoral component to a long porous-coated cementless stems and fixation of the fracture fragment. Revision of the acetabular component if indicated
B3 Fracture around stem or just below it, with proximal bone that is poor quality or severely comminuted. Femoral component revision with proximal femoral allograft or proximal femoral replacement
C Fracture occurs well below the prosthesis ORIF with plate
- leave the hip and acetabular prosthesis alone
A 32-year-old man presents with low back and hip pain that has been gradually worsening over the past year. He reports the symptoms are worse in the morning. Radiographs are shown in Figure A (bilateral sacroilitis) . Laboratory studies show a positive HLA-B27. What additional finding will help confirm the diagnosis?
Erythema marginatum Positive HLA-DR3 Uveitis Positive Rheumatoid Factor Elevated urine phosphoethanolamine
Bilateral sacroiliitis (with or without uveitis) and a postive HLA-B27 is diagnostic of ankylosing spondylitis.
Ankylosing spondylitis is characterized by a positive HLA-B27 with a negative RF titer. It typically presents in the 4th decade of life and is more common in men than women. Low back pain usually precedes the radiogaphic findings of bilateral sacroiliitis. Of note, HLA-B27 is positive in ~6% of the white population.
Rudwaleit et al, looked at variables that could help make an early diagnosis of ankylosing spondylitis. They found the highest likelihood ratio was found in patients with a positive HLA test and positive MRI findings.
Burgos-Vargas et al, studied the clinical and radiographic features of sacroiliac and spinal involvement in patients with seronegative enthesopathy and arthropathy. Based on their findings, they recommend periodical measurements of the spinal flexion and radiographs of the pelvis from age 3 in high risk children.
Incorrect Answers:
Answer 1: Erythema marginatum is a major criteria for Acute Rheumatic Fever.
Answer 2: HLA-DR3 is associated with SLE.
Answer 4: RF is found in rheumatoid arthritis, Sjogren’s, sarcoid, and SLE.
Answer 5: Elevated urine phosphoethanolamine is found in hypophosphatasia.
An systemic chronic autoimmune spondyloarthropathy characterized by ◦HLA-B27 histocompatability complex positive (90%)
◦RF negative (seronegative)
◦primarily affect axial spine
•Pathoanatomy ◦exact mechanism is unknown, but most likely due to an autoimmune reaction to an environmental pathogen in a genetically susceptible individual.
◦theories of relation to HLA-B27 include ◾HLA-B27 aggregates with peptides in the joint and leads to a degenerative cascade
◾cytotoxic T-cell autoimmune reaction against HLA-B27
◦enthesitis ◾entheses inflammation leads to bony erosion, surrounding soft-tissue ossification, and eventually joint ankylosis
◾preferentially targets sacroiliac joints, spinal apophyseal joints, symphysis pubis
◾this differentiates from RA, which is a synovial process
◦disc space involvement ◾inflammation of the annulus lead to bridging osteophyte formation (syndesmophytes)
•Genetics ◦there is a genetic predisposition, but mode of inheritance is unknown
◦HLA-B27 is located on sixth chromosome, B locus
•Epidemiology ◦4:1 male:female
◦affects ~0.2% of Caucasian population
◦usually presents in 3rd decade of life ◾ juvenile form <16-years-old includes enthesitis
◾fewer than 10% of HLA-B27 positive patients have symptoms of AS
•Diagnostic criteria
◦bilateral sacroiliitis
◦+/- uveitis
◦HLA-B27 positive
•Systemic manifestations ◦acute anterior uveitis & iritis
◦heart disease (cardiac conduction abnormalities)
◦pulmonary fibrosis
◦renal amyloidosis
◦ascending aortic conditions (aortitis, stenosis, regurgitation)
◦Klebsilella pneumoniae synovitis (HLA-B27 individuals are more susceptible to Klebsilella pneumoniae synovitis)
•Orthopaedic manifestations ◦bilateral sacroiliitis ◦progressive spinal kyphotic deformity ◦cervical spine fractures ◦large-joint arthritis (hip and shoulder)
Anatomy
•Enthesis ◦defined as the insertion of tendon, ligaments, or muscle into bone
Presentation
•Symptoms
◦lumbosacral pain and stiffness - present in most patients, worse in morning, insidious onset in 3rd decade of life
◦neck and upper thoracic pain - occurs later in life. Acute neck pain should raise suspicion for fracture
◦sciatic: likely originates from sciatic nerve involvement in the pelvic (piriformis spasm)
◦loss of horizontal gaze
◦shortness of breath: caused by costovertebral joint involvement, leading to reduced chest expansion
•Physical exam
◦limitation of chest wall expansion: < 2cm of expansion is more specific than HLA-B27 for making diagnosis
◦decreased spine motion - Schober test used to evaluate lumbar stiffness
◦kyphotic spine deformity: chin-on-chest (flexion) deformity of the spine caused by multiple microfractures that occur over time, chin-brow-to-vertical angle (CBVA) - measured from standing exam of standing lateral radiograph (useful for preoperative planning - correction of this angle correlates with improved surgical outcomes)
◦hip flexion contracture: examining patient in supine and sitting position helps differentiate sagittal plane imbalance due to hip flexion contractures or kyphotic spinal deformity
◦sacroiliac provocative tests: Faber test - flexion abduction external rotation of the ipsilateral hip causes pain
A 23-year-old soccer player suffers an ACL rupture and undergoes reconstruction. Post-operatively she begins a rehabilitation program and her therapist develops a series of knee conditioning exercises to help her regain strength and range of motion. Which of the following exercises places the lowest strain in this patients properly placed ACL graft?
Isometric hamstring contractions at 60 degrees of knee flexion
Isolated quadriceps contractions with the knee at 30 degrees of flexion
Simultaneous quadricep and hamstring contractions at 15 degrees of knee flexion
Isolated quadriceps contractions with the knee at 15 degrees of flexion
Active resisted knee motion from terminal extension to 30 degrees of flexion
Isometric hamstring contractions at 60 degrees of knee flexion will produce the lowest strain in this patient’s ACL graft. Straight leg raises are also commonly used in post-ACL rehabilitation protocols as this exercise places little stress on an ACL graft. The other exercises mentioned have been shown to result in increased graft strain in patients with a reconstructed ACL.
Beynnon et al measured the strain behavior of the ACL during rehabilitation activities in vivo. They found that exercises that produce low or unstrained ligament values, and would not endanger a properly implanted graft, are either dominated by the hamstrings muscle (isometric hamstring contractions at any angle), involve quadriceps muscle activity with the knee flexed at 60 degrees or greater (isometric quadriceps, simultaneous quadriceps and hamstrings contraction), or involve active knee motion between 35 degrees and 90 degrees of flexion.
Rehabilitation
•Early postoperative ◦immediate ◾aggressive cryotherapy (ice)
◾immediate weight bearing (shown to reduce patellofemoral pain)
◾emphasize early full passive extension (especially if associated with MCL injury or patella dislocation)
◦early rehab ◾focus rehab on exercises that do not place excess stress on graft ◾appropriate rehab ◾eccentric strengthening at 3 weeks has been shown to result in increased quadriceps volume and strength
◾isometric hamstring contractions at any angle
◾isometric quadriceps, or simultaneous quadriceps and hamstrings contraction
◾active knee motion between 35 degrees and 90 degrees of flexion
◾emphasize closed chain (foot planted) exercises
◾avoid: isokinetic quadricep strengthening (15-30°) during early rehab
◾open chain quadriceps strengthening
Complications
•Failure due to Tunnel Malposition is the most common cause of ACL failure - causes failure in 70%
◦femoral tunnel malposition
◾coronal plane - vertical femoral tunnel placement caused by starting femoral tunnel at the vertical position in the notch (12 o:clock) as opposed to lateral wall (9 o: clock). Will cause continued rotational instability which can be identified on physical exam by a positive pivot shift
◾sagittal plane - anterior tunnel placement leads to a knee that is tight in flexion and loose in extension - occurs from failure to clear “residents ridge”
◾posterior misplacement (over-the-top), leads to a knee that is lax in flexion and tight in extension
◦tibial tunnel malposition
◾sagittal plane: anterior misplacement leads to knee that is tight in flexion with impingement in extension
◾posterior misplacement leads to an ACL that will impinge with the PCL
•Other cause of failure ◦inadequate graft fixation e.g. can be caused by graft-screw divergence >30 degrees
◦missed diagnosis ◾in combined ACL and PLC injuries, failure to treat the PLC will lead to failure of ACL reconstruction
◦overaggressive rehab
•Infection ◦septic arthritis ◾coagulase negative Staph (S. epidermidis) most common ◾Staph aureus 2nd most common
◾presentation ◾pain, swelling, erythema, and increased WBC at 2-14 days postop
◾treatment ◾perform immediate joint aspiration with gram stain and cultures
◦treatment ◾immediate arthroscopic I&D
◾often can retain graft with multiple I&Ds and antibiotics (6 weeks minimum) ◾graft retention more likely to be successful with S. epidermidis
◾graft retention less likely to be successful with S. aureus
•Loss of motion & arthrofibrosis ◦preoperative prevention ◾be sure patient has regained full ROM before you operate (“pre-hab”)
◾wait until swelling (inflammatory phase) has gone down to reduce the incidence of arthrofibrosis
◦operative prevention ◾proper tunnel placement is critical to have a full range of motion
◦postop prevention ◾aggressive cryotherapy (ice)
◦treatment ◾< 12 weeks, then treat with aggressive PT and serial splinting
◾> 12 weeks, then treat with lysis of adhesions/manipulation under anesthesia
- Infrapatellar contracture syndrome: an uncommon complication following knee surgery or injury which results in knee stiffness. The physical exam will show decreased patellar translation
- Patella Tendon Rupture ◦will see patella alta on the lateral radiograph
- RSD (complex regional pain syndrome)
- Patella fracture ◦most fx occur 8-12 weeks postop
- Hardware failure
- Tunnel osteolysis ◦treat with observation
- Late arthritis ◦related to meniscal integrity
- Local nerve irritation ◦saphenous nerve
- Cyclops lesion : fibroproliferative tissue blocks extension, “click” heard at terminal extension
A 2-year-old child is referred by her pediatrician for fixed flexion deformity of the left thumb. She has been wearing a splint for the last 6 months. She has ventricular septal defect and left renal agenesis. The interphalangeal joint does not extend past 40 degrees of flexion as seen in Figures A and B. There is no triggering. There is a firm, nontender nodule overlying the metacarpophalangeal joint as outlined in blue in Figure C. What is the diagnosis and most appropriate treatment?
Thumb camptodactyly. Therapy including passive stretching exercises.
Congenital clapsed thumb. Percutaneous release of the A1 pulley.
Pediatric trigger thumb. Open release of the A1 pulley.
Pediatric trigger thumb. Open release of the A1 pulley and resection of the tendon nodule.
Blauth Type I hypoplastic thumb. Open release of the A1 pulley and volar plate, and resection of the tendon nodule.
Pediatric trigger thumb. Open release of the A1 pulley.
This child has pediatric trigger thumb (PTT). The potential for spontaneous resolution beyond the age of 2 years is limited. Surgical release of the A1 pulley is indicated.
Pediatric trigger thumb presents as fixed flexion at the interphalangeal joint (IPJ) rather than triggering. It is likely to be acquired (rather than congenital). It is associated with the presence of Notta’s nodule, a thickening of the FPL tendon and overlying tendon sheath. Treatment involves A1 pulley release. The role of non-surgical management (splinting/stretching) remains unclear. The duration of non-surgical treatment is long (up to 30 months) and compliance can be difficult.
Shah et al. reviewed pediatric trigger thumb. The condition is associated with MCP hyperextension. The authors note no advantage to percutaneous release as general anesthetic is required anyway.
Marek et al. performed a retrospective review and survey response review of surgery for pediatric trigger thumb. They found that age at the time of surgery influences residual flexion contracture and rate of recovery. They found surgery to be safe and effective, and recommend: (1) surgery for a 2-year-old child with a locked thumb for 6 months, (2) observation for a child <1 year if the thumb is triggering (not locked), and (3) a 6-month observation period if observation is advocated.
Figures A and B show a fixed flexion deformity of the thumb and an attempt at thumb extension. Figure C shows the outlined Notta nodule.
Incorrect Answers:
Answer 1: Camptodactyly is a congenital flexion deformity that usually involves the little finger. Surgery is indicated in congenital trigger thumb for patients >3 yrs, and those that have failed a course of splinting/stretching. The efficacy of passive exercises has not been established.
Answer 2: Percutaneous release of the A1 pulley in PTT carries a risk of incomplete release, radial digital nerve injury, and FPL laceration. The gold standard is open release. This is not congenital clasped thumb.
Answer 4: The tendon nodule, or Notta’s nodule, does not have to be resected.
Answer 5: The volar plate does not have to be released. This is not thumb hypoplasia. Type I thumb hypoplasia is treated nonoperatively.
A 25-year-old marathon runner presents with pain, coolness, and tingling in her lower leg and foot which are exacerbated with walking, but relieved once she starts running for a few minutes. Compartment pressures are normal at rest and with exercise. Her physical exam is significant for pain with passive dorsiflexion and plantar flexion of the ankle. These symptoms are most consistent with which of the following conditions?
Lumbar radiculopathy Piriformis syndrome Exertional compartment syndrome Popliteal artery entrapment syndrome Tibial stress fracture
The clinical presentation is most consistent with popliteal artery entrapment syndrome.
Patients with popliteal artery entrapment syndrome typically present with intermittent claudication and decreased pulses. The pathoanatomy involves compression of the popliteal artery by the medial head of the gastrocnemius. Therefore, dorsiflexing and plantarflexing the ankle are provocative maneuvers that would worsen the claudication as a result of the compressive effect of the medial gastrocnemius on the popliteal artery. An arteriogram would demonstrate the popliteal artery compression by the medial head of the gastrocnemius.
Gokkus et al. present a case and systematic review of popliteal artery entrapment syndrome. They state the differential diagnosis for patients with lower leg pain from exercise includes chronic exertional compartment syndrome, medial tibia stress syndrome, fibular and tibial stress fractures, fascial defects, nerve entrapment syndrome, vascular claudication (artherosclerotic or popliteal artery entrapment syndrome) and lumbar disc herniation.
Illustration A shows MR angiogram images demonstrating a segmental occlusion of the right popliteal artery.
Incorrect Answers:
Answer 1&2: Since the patient is not exhibiting signs of a lower extremity nerve entrapment syndrome (neuropathy would not explain the cool crampy leg), lumbar radiculopathy and piriformis syndrome should not be the 1st choice (in this case the parasthesias are a result of the claudication).
Answer 3: Elevated post exercise compartment pressure measurements are essential to diagnose exertional compartment syndrome where vigorous athletic exercise leads to swelling of myofascial compartments in the lower leg leading to pain which is relieved with rest.
Answer 5: A stress fracture would not be relieved with running and would not have exhibited signs of claudication like the cool leg, cramping and parasthesias.
A 25-year-old patient presents with a posterior wall/ posterior column acetabular fracture. She is scheduled for open reduction internal fixation through a posterior approach. What position of the leg exerts the least amount of intraneural pressure on the sciatic nerve?
hip flexion, knee extension hip extension, knee extension hip flexion, knee flexion hip extension, knee flexion the pressure does not vary based on position
hip extension, knee flexion
In the cited study, researchers measured tissue fluid pressure within the sciatic nerve in cadaveric specimens using a pressure transducer. The hip and knee were taken through a combination of ranges and found that the clinically relevant increase in pressure happened with the hip flexed at 90 degrees and the knee fully extended. They concluded that increased intraneural pressure was related to excursion of the nerve as linear distance between the greater sciatic notch and the distal leg increase. Hence, according to the question stem, to avoid traction injury, the reverse position should be implemented (hip extension and knee flexion).
During the early swing phase of the normal gait cycle, what lower extremity muscle is primarily contracting?
Tibialis posterior Tibialis anterior Vastus medialis Adductor longus Gastrocnemius
Tib ant
Electromyography during walking reveals the tibialis anterior muscle is active during early swing, allowing the foot to clear the ground. All of the other muscles are quiet, as the limb moves forward through space with minimal muscular effort. The other muscles are primarily active during weight acceptance or push-off.
•One gait cycle is measured from heel-strike to heel-strike
Consists of :
◾stance phase: period of time that the foot is on the ground, ~60% of one gait cycle is spent in stance. during stance, the leg accepts body weight and provides single limb support
◾swing phase: period of time that the foot is off the ground moving forward, ~40% of one gait cycle is spent in swing - the limb advances
- Stride: the distance between consecutive initial contacts of the same foot with the ground
- Step: the distance between initial contacts of the alternating feet
Swing Phase
•Initial swing (toe off) - start of single limb support for opposite limb
◦definition: from elevation of limb to point of maximal knee flexion
◦muscular contractions: hip flexors concentrically contract to advance the swinging leg
•Mid-swing (foot clearance)
◦definition ◾following knee flexion to point where tibia is vertical
◦muscular contractions: ankle dorsiflexors contract to ensure foot clearance
•Terminal swing (tibia vertical) ◦definition ◾from point where tibia is vertical to just prior to initial contact
◦muscular contractions ◾hamstring muscles decelerate forward motion of thigh
Variables Affected During Gait Cycle
•Pelvic rotation ◦pelvis rotates 4 degrees medially (anteriorly) on swing side ◾lengthens the limb as it prepares to accept weight
•Pelvic tilt ◦pelvis drops 4 degrees on swing side ◾lowers COG at midstance
•Knee flexion in stance: early knee flexion (15 degrees) at heel strike ◾lowers COG, decreasing energy expenditure
◾also absorbs shock of heel strike
•Foot mechanisms: ankle plantar flexion at heel strike and first part of stance
•Knee mechanisms ◦at midstance, the knee extends as the ankle plantar flexes and foot supinates
◦restores leg to original length
◦reduces fall of pelvis at opposite heel strike
•Lateral displacement of pelvis ◦pelvis shifts over stance limb ◾COG must lie over base of support (stance limb)
•Center of gravity (COG): in standing position is 5cm anterior to S2 vertebral body
◦vertical displacement: during gait cycle COG displaces vertically in a rhythmic pattern. The highest point is during midstance phase and lowest point occurs at the time of double limb support
◦horizontal displacement ◾COG displaces 5cm horizontally during adult male step
A 3-year-old boy presents with the skin lesion seen in Figure A and a leg deformity. Radiographs are shown in Figure B (anterior tibial bowing, no # or pseudoarthrosis). What is the most appropriate first step in treatment?
observation
bracing in total contact orthosis
intramedullary nailing with bone grafting
free fibular graft from contralateral side
external fixation using Illizarov techniques
bracing in total contact orthosis
The clinical presentation is consistent with neurofibromatosis with associated anterolateral tibial bowing. Neurofibromatosis is the most common cause of anterolateral tibial bowing and congenital pseudoarthrosis of the tibia, two conditions which represent a continuum of the same disease process. Because there is no fracture or pseudoarthrosis the treatment in this case is bracing in a total contact orthosis, otherwise known as a clamshell orthosis. If the patient had a fracture or a pseudoarthrosis, then you would treat him with surgery. The reference by Crawford et al is a review article that describes the incidence and treatment of orthopaedic conditions seen with neurofibromatosis. In their database of 588 patients they found the incidence of spinal deformity in children with NF-1 to be 21%; pectus deformity, 4.3%; limb-length inequality, 7.1%; congenital tibial dysplasia, 5%; hemihypertrophy, 1.4%; and plexiform neurofibromas, 25%. The cited reference by Feldman et al is a more recent review article that also discusses the orthopaedic manifestations of neurofibromatosis. There are various tibial bowing conditions found in children and one should be familiar with the differential diagnosis and associated conditions. Bowing of the tibia that is present at birth typically occurs either anteriorly (in association with fibular hemimelia), anterolaterally (in association with congenital pseudoarthrosis), or posteromedially (in association with calcaneovalgus foot deformity).
•Neurofibromatosis is an autosomal dominant disorder of neural crest origin characterized by:
◦extremity deformities
◾congenital anterolateral bowing and pseudoarthrosis of tibia/ fibula and forearm
◾hemihypertrophy
◦spine involvement ◾scoliosis & kyphosis
◾atlantoaxial instability
•Epidemiology ◦ 1:3,000 births for NF1
•Genetics ◦autosomal dominant (AD)
◦mutation in NF1 gene on chromosome 17q21 ◾codes for neurofibromin protein ◾negatively regulates Ras signaling pathway
◾neurofibromin deficiency leads to increased Ras activity
◾affects Ras-dependent MAPK activity which is essential for osteoclast function and survival
◦neurofibromatosis is the most common genetic disorder caused by a new mutation of a single gene
•Associated conditions ◦scoliosis ◦anterolateral bowing of tibia ◦bowing of forearm bones with obliteration of medullary cavity: ulnar pseudoarthrosis, radius pseudoarthrosis ◦neoplasias
•Prognosis ◦normal life expectancy
◦high incidence of malignancy and hypertension
•Diagnostic criteria ◦according to the NIH Consensus Development Conference Statement (1987) the diagnostic criteria for NF-1 are met in an individual if two or more of the following are found
◾six or more café-au-lait macules over 5 mm in greatest diameter in prepubertal individuals and over 15 mm in postpubertal individuals.
◾two or more neurofibromas of any type or one plexiform neurofibroma.
◾freckling in the axillary or inguinal region.
◾optic glioma.
◾two or more Lisch nodules (iris hamartomas).
◾a distinctive osseous lesion such as sphenoid dysplasia or thinning of long bone cortex with or without pseudarthrosis.
◾a first-degree relative (parent, sibling, or offspring) with NF-1 by the above criteria.is based on presence of both
Classification
•NF1 (von Recklinghaussen disease) ◦most common
•NF2 ◦associated with bilateral vestibular schwannomas
•Segmental NF ◦features of NF1 but involving a single body segment
Presentation ◦often presents with anterolateral bowing of tibia or radial bowing
•Physical exam ◦verrucous hyperplasia
◦hemihypertrophy
◦cafe-au-lait spots
◦axillary freckling
◦scoliosis
◦anterolateral bowing or pseudoarthrosis of tibia
◦dermal Plexiform-type neurofibroma may be seen
◦Lisch nodules are benign pigmented hamartomas of the iris
Anterolateral Tibial Bowing (Neurofibromatosis)
◾anterolateral bowing is often associated with neurofibromatosis (NF1)
◾50% with anterolateral bowing have NF1
◾10% of NF1 have anterolateral bowing
◾may progress to pseudoarthrosis
◦differentials for tibia bowing ◾anteromedial: associated with fibular hemimelia and congenital loss of lateral rays of the foot
◾posteromedial: usually congenital due to abnormal intrauterine positioning - dorsiflexed foot pressed against anterior tibia. Will develop leg length discrepancy - associated with calcaneovalgus deformity
•Imaging ◦radiographs ◾obtain AP and lateral of tib/fib
•Treatment
◦nonoperative: bracing in total contact orthosis
◾indications: bowing without pseudoarthrosis or fracture (goal is to prevent further bowing and fractures)
◾spontaneous remodeling is not expected
◾osteotomy for bowing alone is contraindicated
◦operative: bone grafting with surgical fixation ◾indications: bowing with pseudoarthrosis or fracture
◾amputation with prosthesis fitting ◾indications: Three failed surgical attempts
◾Syme’s often superior to BKA due to atrophic and scarred calf muscle in these patients
•Techniques ◦intramedullary nailing with bone grafting
◾resect pseudoarthrosis
◾insert Charnley-Williams rod
◾antegrade through resection site, then retrograde through the heel
◾< 4 y.o., extend fixation to calcaneus
◾5-10 y.o., extend fixation to talus ◾2 yrs. postop, typically a 2nd surgery to push rod proximally to free the ankle joint
◦free fibular graft ◾often need to take fibula from contralateral side because ilpsilateral fibula is not normal
◾Illizarov’s external fixation
During total hip arthroplasty (THA) via a posterior approach, where is the sciatic nerve most likely to be found?
Superficial to the piriformis and superficial to the short external rotators
Superficial to the piriformis and deep to the short external rotators
Deep to the piriformis and deep to the short external rotators
Deep to the piriformis and superficial to the short external rotators
Splits the piriformis and is superficial to the short external rotators
Deep to the piriformis and superficial to the short external rotators
During the posterior approach to the hip, the most predictable course of the sciatic nerve is deep to the piriformis and superficial to the short external rotators exiting above the superior gemellus. As such, most recommend identification of the sciatic nerve by palpation in primary THA. In revision THA, many advocate identification of the sciatic nerve by both palpation and direct visualization.
The most common anatomic variant in the relationship of the short external rotators and the sciatic nerve is with the sciatic nerve traveling between the capsule and the short external rotators exiting below the superior gemellus.
Smoll reviewed the anatomy of the gluteal region and sciatic nerve anomalies in a meta-analysis and review of over 6000 cadavers. They concluded that the anomalies were present in about 16.8% of cadavers. They recommended a heightened awareness of the anomalies in hip surgery. The most common variants are found in Illustration D which were also supported by an earlier Beaton et al study.
A 52-year-old male farmer presents with right hip pain for the past 4 months. On physical examination there is pain with internal rotation of his right hip. Laboratory studies show elevated serum alkaline phosphatase. Serum calcium is normal. Urinary studies show elevated urinary N-telopeptide, alpha-C-telopeptide, and deoxypyridinoline markers. A radiograph of the pelvis is shown in Figures A. What would be the most appropriate treatment for this patient?
Antibiotics Bisphosphonates Monoclonal-B antibodies Chemotherapy Wide resection and radiotherapy
This patient has the clinical presentation of symptomatic Paget’s disease. The most appropriate initial treatment would be medical management with bisphosponate therapy.
Paget’s disease is the second most common chronic bone-remodelling disorder in the US after osteoporosis. Elevated serum alkaline phosphatase, normal serum calcium and elevated urinary N-telopeptide, alpha-C-telopeptide, and deoxypyridinoline markers, are indicative of Paget’s disease. First line treatment for symptomatic Paget’s disease would include medical management for osteoclast inhibition, including bisphosphonates or calcitonin, as well as supportive symptomatic therapy with physiotherapy, NSAIDS, or oral analgesics.
Hadjipavlou et al. reviewed Paget’s disease of the bone and its management. Farmers have been shown to have an increased incidence of Paget’s disease. Average age at presentation is in the 5th and 6th decade. Symptomatic individuals are recommended to be treated initially with medical management. They suggest that bisphosphonates are more effective than calcitonin at suppressing the histological and biochemical activity in Paget’s disease.
Figure A shows thickening of the cortex and a coarse trabecular pattern in right hemipelvis characteristic of Paget’s bone disease. There are 4 radiographic phases of Paget’s bone disease: the osteolytic, osteoblastic, osteosclerotic and mixed phase. This radiograph shows mixed phase. Note significant arthritis of the right hip joint.
Incorrect Answers:
Answer 1: The history, laboratory and radiographic findings are not suggestive of osetomyelitis
Answer 3: Immunotherapy is not used in the treatment of Paget’s disease
Answer 4: Chemotherapy is not used in the initial treatment of Paget’s disease
Answer 5: Wide resection and radiotherapy is not used in the initial treatment of Paget’s disease
•A condition of abnormal bone remodeling : original osseous tissue is reconstructed through active interplay between excessive bone resorption and abnormal new bone formation
•Pathophysiology
◦increased osteoclastic bone resorption is the primary cellular abnormality - cause is thought to be a slow virus infection (intra-nuclear nucleocapsid-like structure) ◾paramyxovirus
◾respiratory syncytial virus
•Epidemiology ◦peak incidence in the 5th decade of life ◦common in Caucasians (northern European / Anglo-Saxon descent)
◦males = females
◦location - may be monostotic or polyostotic
◾common sites include femur > pelvis > tibia > skull > spine
•Genetics: most cases are spontaneous
◾hereditary: familial clusters have been described with ~40% autosomal dominant transmission
◾most important is 5q35 QTER (ubiquitine binding protein sequestosome 1) SQSTM1 (p62/Sequestosome) - tend to have severe Paget disease
◾also insertion mutation in TNFRSF11A for gene encoding RANK
•Orthopaedic manifestations
◦bone pain
◦long bone bowing
◦fractures, due to brittle bone and tend to be transverse
◦large joint osteoarthritis - excessive bleeding during THA, malalignment during TKA
◦secondary sarcoma
•Associated conditions
◦high output heart failure
•Prognosis & malignancy
◦Paget’s sarcoma: less than 1% will develop malignant Paget’s sarcoma (secondary sarcoma) ◾osteosarcoma > fibrosarcoma and chondrosarcoma - most common in pelvis, femur, and humerus. Poor prognosis: 5-year survival for metastatic Paget’s sarcoma < 10%. Treatment includes chemotherapy and wide surgical resection
Classification
•Phases
◦lytic phase: intense osteoclastic resorption
◦mixed phase: resorption and compensatory bone formation
◦sclerotic phase: osteoblastic bone formation predominates
◦all three phases may co-exist in the same bone
Presentation
•Symptoms
◦asymptomatic: frequently asymptomatic and found incidentally
◦pain ◾pain may be the presenting symptom due to ◾stress fractures
◾increased vascularity and warmth
◾new intense pain and swelling ◾suspicious for Paget’s sarcoma in a patient with history of Paget’s + new intense pain and swelling
◦cardiac symptoms ◾can present with high-output cardiac failure particularly if large/multiple lesions & pre-existing diminished cardiac function
Imaging
•Radiographs: coarsened trabeculae which give the bone a blastic appearance - both increased and decreased density may exist depending on phase of disease
◾lytic phase: lucent areas with expansion and thinned, intact cortices, ‘blade of grass’ or ‘flame-shaped’ lucent advancing edge
◾mixed phase: combination of lysis + sclerosis with coarsened trabeculae
◾sclerotic phase: bone enlargement with cortical thickening, sclerotic and lucent areas image
◦remodeled cortices: loss of distinction between cortices and medullary cavity
◦long bone bowing : bowing of femur or tibia image
◦fractures
◦hip and knee osteoarthritis
◦osteitis circumscripta - (cotton wool exudates) in skull image
◦Paget’s secondary sarcoma: shows cortical bone destruction, soft tissue mass
•MRI: may show lumbar spinal stenosis
•Bone scan image ◦accurately marks site of disease:
◦intensely hot in lytic and mixed phase
◦less hot in sclerotic phase
•CT scan: cortical thickening and coarsened trabeculae
Evaluation
•Laboratory findings
◦elevated serum ALP
◦elevated urinary collagen cross-links
◦elevated urinary hydroxyproline (collagen breakdown marker)
◦increased urinary N-telopeptide, alpha-C-telopeptide, and deoxypyridinoline
◦normal calcium levels
Histology
•Characteristic histology: woven bone and irregular broad trabeculae with disorganized cement lines in a mosaic pattern
◦profound bone resorption - numerous large osteoclasts with multiple nuclei per cell - virus-like inclusion bodies in osteoclasts. Paget’s osteoclasts larger, more nuclei than typical osteoclasts
Fibrous vascular tissue interspersed between trabeculae
Treatment
•Nonoperative: observation and supportive therapy. Treatment for asymptomatic Paget’s disease: physiotherapy, NSAIDS, oral analgesics
◦medical therapy aimed at osteoclast inhibition: bisphosphonates are 1st line treatment for symptomatic Pagets
◾oral alendronate and risedronate
◾etidronate disodium (Didronel) (older generation medication, inhibits osteoclasts and osteoblasts, cannot be used for more than 6 months at a time)
◾intravenous: pamidronate, zoledronic acid (Zometa) - newer generation medications that only inhibit osteoclasts
◾calcitonin are 2nd line (after bisphosphonates): causes osteoclasts to shrink in size and decreases their bone resorptive activity within minutes- administered subcutaneously or intramuscularly
◾teriparatide is contraindicated in Paget’s disease due to risk of secondary osteosarcoma
•Operative ◦THA / TKA ◾indications ◾affected patients with degenerative joint disease. Treat Paget’s with pharmacologic agents prior to arthroplasty to reduce bleeding
◾outcomes: greater incidence of suboptimal alignment secondary to pagetoid bone. The most common complications include malalignment with knee arthroplasty and bleeding with hip arthroplasty
◦metaphyseal osteotomy and plate fixation
◾indications:
◾fractures through pathologic bowing of long bones
◾impending pathologic fracture of long bone with bowing
All of the following are true regarding excessively anterior femoral tunnel placement during ACL reconstruction EXCEPT?
It may cause loss of knee flexion
It may cause graft over-stretching and failure
It is the most common technical error
It may cause interference screw divergence
It is often due to poor visualization
All of the given responses are true except for Answer 4, because an excessively anterior femoral tunnel does not cause interference screw divergence.
Anterior placement of the femoral tunnel is the most common surgical error during arthroscopic ACL reconstruction. Errors in surgical technique are one of the most common reasons for graft failure in patients who present with recurrent instability after ACL reconstruction. Technical shortcomings that result in graft failure after primary reconstruction include nonanatomic tunnel placement, graft impingement, improper tensioning of the graft, inadequate fixation of the graft in bony tunnels, graft material problems, and the failure to address insufficiency of the secondary stabilizers of the knee during ACL reconstruction.
The papers by Allen and Harner et al stress the importance of determining the cause of failure prior to revision surgery. It is estimated that 70% to 80% of graft failures are caused by malpositioned tunnels. The consequences of nonanatomic tunnel placement are well described in the literature.
Sommer et al note that inappropriate positioning of either the tibial or femoral tunnels results in excessive changes in graft length as the knee moves through its functional range of motion and can effect clinical results. Because biologic ACL grafts can only accommodate small changes in length before undergoing plastic deformation, a mal-positioned graft may result in either capturing of the knee or lengthening of the graft over time; this results in either a loss of motion or recurrent instability, respectively. Improper femoral tunnel placement is most often caused by the failure to adequately visualize the most posterior aspect of the notch (the “over-the-top” position). Because the femoral attachment of the ACL is closer to the center of rotation of the knee, small errors in femoral tunnel placement may have deleterious effects on knee kinematics.
Complications of ACL Repair:
Complications
•Failure due to Tunnel Malposition is the most common cause of ACL failure - causes failure in 70%
◦femoral tunnel malposition
◾coronal plane - vertical femoral tunnel placement caused by starting femoral tunnel at the vertical position in the notch (12 o:clock) as opposed to lateral wall (9 o: clock). Will cause continued rotational instability which can be identified on physical exam by a positive pivot shift
◾sagittal plane - anterior tunnel placement leads to a knee that is tight in flexion and loose in extension - occurs from failure to clear “residents ridge”
◾posterior misplacement (over-the-top), leads to a knee that is lax in flexion and tight in extension
◦tibial tunnel malposition
◾sagittal plane: anterior misplacement leads to knee that is tight in flexion with impingement in extension
◾posterior misplacement leads to an ACL that will impinge with the PCL
•Other cause of failure ◦inadequate graft fixation e.g. can be caused by graft-screw divergence >30 degrees
◦missed diagnosis ◾in combined ACL and PLC injuries, failure to treat the PLC will lead to failure of ACL reconstruction
◦overaggressive rehab
•Infection ◦septic arthritis ◾coagulase negative Staph (S. epidermidis) most common ◾Staph aureus 2nd most common
◾presentation ◾pain, swelling, erythema, and increased WBC at 2-14 days postop
◾treatment ◾perform immediate joint aspiration with gram stain and cultures
◦treatment ◾immediate arthroscopic I&D
◾often can retain graft with multiple I&Ds and antibiotics (6 weeks minimum) ◾graft retention more likely to be successful with S. epidermidis
◾graft retention less likely to be successful with S. aureus
•Loss of motion & arthrofibrosis ◦preoperative prevention ◾be sure patient has regained full ROM before you operate (“pre-hab”)
◾wait until swelling (inflammatory phase) has gone down to reduce the incidence of arthrofibrosis
◦operative prevention ◾proper tunnel placement is critical to have a full range of motion
◦postop prevention ◾aggressive cryotherapy (ice)
◦treatment ◾< 12 weeks, then treat with aggressive PT and serial splinting
◾> 12 weeks, then treat with lysis of adhesions/manipulation under anesthesia
- Infrapatellar contracture syndrome: an uncommon complication following knee surgery or injury which results in knee stiffness. The physical exam will show decreased patellar translation
- Patella Tendon Rupture ◦will see patella alta on the lateral radiograph
- RSD (complex regional pain syndrome)
- Patella fracture ◦most fx occur 8-12 weeks postop
- Hardware failure
- Tunnel osteolysis ◦treat with observation
- Late arthritis ◦related to meniscal integrity
- Local nerve irritation ◦saphenous nerve
- Cyclops lesion : fibroproliferative tissue blocks extension, “click” heard at terminal extension
A 10-year-old boy presents with a painless mass on the dorsal aspect of his wrist that has been present for 3 weeks. A clinical image is shown in Figure A (well circumscribed, transillumates, not fixed to skin). T1 and T2 magnetic resonance images are shown in Figure B and C (well-marginated, homogenous signal intensity mass) , respectively. On your exam, the mass transilluminates and Allen test reveals patent radial and ulnar arteries. What is the most appropriate next step in management?
Referral to a orthopaedic oncologist Surgical excision with wide margins Observation Autologus bone marrow aspirate injection Injection of N-Butyl-Cyanoacrylate
This child has a ganglion cyst on the dorsal aspect of his wrist. Imaging provided shows a well-marginated, homogenous signal intensity mass consistent with a ganglion cyst. Physical examination findings of a mass transilluminating corroborate the MRI findings of a ganglion cyst. Performing an Allen’s test to evaluate radial and ulnar artery collateral blood flow is especially important when evaluating ganglion cysts on the volar aspect of the wrist as they are often adjacent to the radial artery. Wang et al. peformed a Level 4 review of 14 children with hand and wrist ganglion cysts and found that 79% of these cysts resolved spontaneously within 1 year. Autologus bone marrow aspirate injection is a treatment option for unicameral bone cysts and N-Butyl-Cyanoacrylate injections have been described for treatment of hemangiomas. Referral to an orthopaedic oncologist is not indicated.
A mucin-filled synovial cyst caused by either trauma, mucoid degeneration, synovial herniation
incidence: it is the most common hand mass (60-70%)
location:
dorsal carpal (70%) - originate from SL articulation
volar carpal (20%) - originate from radiocarpal or STT joint
volar retinacular (10%) - originate from herniated tendon sheath fluid
dorsal DIP joint (mucous cyst, associated with Heberden’s nodes)
Pathophysiology
filled with fluid from tendon sheath or joint, no true epithelial lining
Associated conditions
median or ulnar nerve compression may be caused by volar ganglion
hand ischemia due to vascular occlusion may be caused by volar ganglion
Presentation
usually asymptomatic - may cause issues with cosmesis
Physical exam
inspection: transilluminates (transmits light through tissue)
palpation, firm and well circumscribed, often fixed to deep tissue but not to overlying skin
Vascular exam
Allen’s test to ensure radial and ulnar artery flow for volar wrist ganglions
Imaging
Radiographs: normal
MRI - not routinely indicated. Shows well marginated mass with homogenous fluid signal intensity
Ultrasound: useful for differentiating cyst from vascular aneurysm - may provide image localization for aspiration while avoiding artery
Histology
Biopsy - not routinely indicated, will show mucin-filled synovial cell lined sac
Treatment
Nonoperative
observation: first line of treatment in adults, children - 76% resolve within 1 year in pediatric patients
closed rupture: home remedy, high recurrence
Aspiration: second line of treatment in adults with dorsal ganglions. Aspiration typically avoided on volar aspect of wrist due to radial artery. Higher recurrence rate (50%) than surgical resection but minimal risk so reasonable to attempt
Operative: surgical resection
indications: severe symptoms or neurovascular manifestations
Requires adequate exposure to identify origin and allow resection of stalk and a portion of adjacent capsule. At dorsal DIP joint: must resect underlying osteophyte
results
volar ganglions have higher recurrence after resection than dorsal ganglions (15-20% recurrence)
Complications
With aspiration: infection (rare), neurovascular injury
With excision: infection, neurovascular injury (radial artery most common), injury to scapholunate interosseous ligament, stiffness
A 45-year-old male falls onto his left shoulder while biking and an injury radiograph is shown in Figure A (comminuted displaced mid shaft clavicle fracture). He elects for nonoperative treatment. What is the most likely clinical outcome at one year after injury?
Symmetric cosmesis of shoulders Decreased shoulder motion Symptomatic nonunion Shoulder instability Decreased shoulder strength and endurance
Patients who have nonoperative treatment of displaced midshaft clavicle fractures have significant decreases in both strength and endurance to approximately 80% of the contralateral side as described by the McKee article. There was a trend correlating shortening >2cm with poor outcome (p=0.06). Motion was found to be preserved.
In the Canadian Orthopaedic Trauma Society’s landmark randomized control trial of operative versus nonoperative treatment for displaced clavicle fractures, patients treated non-operatively had lower subjective outcomes scores, slower rates to union, more nonunions, more symptomatic malunions, and were less satisfied with the appearance of their shoulder. There were more hardware related complications in the operatively treated group.
The second McKee article describes improvements in subjective outcome scores after midshaft clavicle malunion corrective osteotomy.
Figure A shows a comminuted, displaced midshaft clavicle fracture.
Clavicle shaft fractures are common traumatic injuries that occur in the middle third of the clavicle - treatment is somewhat controversial but may be nonoperative or operative based on the degree of displacement and patient factors
incidence
clavicle fractures account for 2.6-4% of all adult fractures
demographics
often seen in young, active patients - most common in males < 30 years old
Pathophysiology
mechanism of injury: fall on an outstretched arm or direct trauma to the shoulder
Pathoanatomy
75-80% of all clavicle fractures will occur in the middle third segment- the junction of the outer and middle thirds is the thinnest part of the bone and is the only area not protected by or reinforced with muscle and ligamentous attachments
it is therefore prone to fracture, particularly with axial loading
displaced fractures:
- medial fragment: sternocleidomastoid muscle pulls the medial fragment posterosuperiorly
- lateral fragment: pectoralis and weight of arm pull the lateral fragment inferomedially
- open fractures usually the result of the medial fragment as it “buttonholes” through the platysma
Associated conditions - rare but may include
ipsilateral scapular fracture
scapulothoracic dissociation (should be considered with significantly displaced/widened fracture fragments)
rib fracture
pneumothorax
neurovascular injury
Treatment
Nonoperative: sling immobilization or figure of 8 brace with gentle ROM exercises at 2-4 weeks and strengthening at 6-10 weeks
indications
< 2cm shortening and displacement
< 1cm displacement of the superior shoulder suspensory complex
no neurovascular injury
outcomes: nonunion (1-5%)
risk factors: comminution, > 100% displacement, > 2cm shortening, advanced age, female gender, poorer cosmesis, decreased shoulder strength and endurance - seen with displaced midshaft clavicle fractures healed with > 2cm of shortening
Operative
closed reduction and intramedullary fixation vs. open reduction internal fixation
indications
- absolute: open fractures, displaced fracture with skin tenting, subclavian artery or vein injury, floating shoulder (clavicle and scapular neck fracture), symptomatic nonunion, symptomatic malunion
- relative and controversial indications: displaced with > 2cm shortening, bilateral displaced clavicle fractures, brachial plexus injury (questionable because 66% have spontaneous return), closed head injury, seizure disorder, polytrauma patient
Closed Reduction and Intramedullary Fixation
contraindications: substantial comminution, segmental fractures
approach: beach chair or supine, posterolateral incision
instrumentation: cannulated screw, specialized screw systems (e.g, Dual Trak), titanium elastic nail, Hagle pin
advantages: smaller incision, less soft-tissue disruption, less prominent hardware, avoids the supraclavicular cutaneous nerves commonly injured with plating
disadvantages: higher complication rate including hardware migration, hardware breakage, temporary brachial plexus palsy, and skin breakdown over the entry portal. Biomechanically inferior to plating
Open Reduction Internal Fixation
approach: beach chair vs. supine, direct superior vs. anterior incision
instrumentation
most common: limited contact, pre-controured, dynamic compression plate with k-wires for preliminary fixation
other options: 3.5mm reconstruction plate, locking plates
technique: superior vs. anteroinferior plating
higher load to failure (superior plating > anterointerior plating)
decreased plate strength with inferior bone comminunion (anteroinferior plating > superior plating)
lower risk of neurovascular injury (anteroinferior plating > superior plating)
lower removal of deltoid attachment (superior plating > anterointerior plating)
advantages: improved results with ORIF for clavicle fractures with > 2cm shortening and > 100% displacement
improved functional outcomes/less pain with overhead activity, faster time to union, decreased symptomatic malunion rate, improved cosmetic satisfaction, improved overall shoulder satisfaction, increased shoulder strength and endurance
disadvantages: increased risk of need for future procedures- implant removal, debridement for infection
outcomes: time to union - operative (16.4 weeks) vs. non-operative (28.4 weeks)
Postoperative Rehabilitation
early: sling for 7-10 days followed by active motion
late
strengthening at ~6 weeks when pain-free motion and radiographic evidence of union
full activity including sports at ~3 months
Complications of Nonoperative treatment: nonunion (1-5%)
risk factors: fracture comminution (Z deformity), fracture displacement, female gender, advanced age, smoker
treatment
if asymptomatic, no treatment necessary
if symptomatic, ORIF with plate and bone graft (particularly atrophic nonunion)
malunion - definition: shortening > 3cm, angulation > 30°, translation > 1cm
presentation
increased fatigue with overhead activities
thoracic outlet syndrome
dissatisfaction with appearance
difficulty with shoulder straps, backpacks and the like
treatment
clavicle osteotomy with bone grafting, if symptomatic
Operative treatment
hardware prominence ~30% of patient request plate removal
superior plates associated with increased irritation
neurovascular injury (3%)
superior plates associated with increased risk of subclavian artery or vein penetration
subclavian thrombosis
nonunion (1-5%)
infection (~4.8%)
risk factors: illicit drug use, diabetes, previous shoulder surgery, mechanical failure (~1.4%), pneumothorax, adhesive capsulitis - 4% in surgical group develop adhesive capsulitis requiring surgical intervention
A 62-year-old gentleman with a 10-year history of Type II diabetes complains of warmth, swelling, and pain in his right foot that has progressively worsened over the past 6 weeks. He denies fevers or chills, and states that the swelling and warmth dissipates each night after he sleeps with his foot elevated on pillows. A clinical photograph of the foot is provided in Figure A. The midfoot is hot to touch and mildly tender with palpation. A radiograph is provided in Figure B. Which of the following is the most appropriate management?
Custom orthotics with first ray recession and lateral heel posting
Total contact cast and non-weight bearing
Intravenous antibiotics
Talonavicular and tarsometarsal arthrodeses
Transtibial amputation
The clinical presentation, photograph, and radiograph are consistent with diabetic charcot neuropathy of the midfoot.
The lack of systemic symptoms and resolution of erythema with foot elevation rule against the presence of infection. This patient appears to be in the fragmentation phase of the pathologic process given the osteopenia, fracture, and collapse of the midfoot. This is followed by the coalescence and reconstitution phases. The goal of treatment in the first phase is to prevent further collapse and deformity. Given there is no ulceration, and/or deep infection present, this is best accomplished through protected weight bearing and total contact casting. A CROW walker could also be considered.
Examples of total contact casting and a CROW walker are demonstrated in Illustrations A and B. The objective is to prevent high contact stress points and subsequent ulceration in the insensate foot. Surgical intervention should be reserved for debridement of a deep infection from open wounds or arthrodesis of arthritic joints following the consolidation that occurs during the reconstitution phase.
Symptoms
swollen foot and ankle
pain in 50%, painless in 50%
loss of function
Physical exam
acute Charcot neuropathy: swollen , warm, average of 3.3 degrees C warmer than contralateral side, erythema - often confused with infection but erythema will decrease with elevation in Charcot arthropathy, but is unchanged in infection
chronic Charcot neuropathy: structurally deformed foot
bony prominences, rocker bottom deformity , collapse of medial arch,
motion: may be ligamentously unstable
neurovascular: Semmes-Weinstein monofilament testing
Radiographs
obtain standard AP and lateral of foot, complete ankle series
early changes: degenerative changes may mimic osteoarthritis
late changes: obliteration of joint space, fragmentation of both articular surfaces of a joint leading to subluxation or dislocation, scattered “chunks” of bone in fibrous tissue surrounding soft tissue edema, joint distension by fluid
heterotopic ossification
Bone scan - useful to help determine presence of superimposed osteomyelitis
type of study
technetium bone scan: may be positive for a neuropathic joint and osteomyelitis
indium WBC scan: negative (cold) for neuropathic joints and positive (hot) for osteomyelitis
MRI : best for differentiating abscess from soft-tissue swelling - most sensitive in diagnosing soft tissue and/or osteomyelitis
limitations - difficult to differentiate infection from Charcot arthropathy on MRI
A 26-year-old football offensive lineman presents with shoulder pain which is affecting his ability to block effectively. On exam, he has a positive jerk test and a positive Kim test. What is his most likely diagnosis?
SLAP tear ALPSA lesion Hill-Sachs lesion Posterior labral tear PASTA tear
The posterior jerk test is a sensitive exam for ascertaining the presence of posterior glenoid labral tears in the mid-range of the glenoid. The Kim test is more sensitive for posterior-inferior labral tears. This is performed by having the patient seated, arm at 90° abduction, followed by flexing the shoulder to 45 forward flexion while simultaneously applying axial load on the elbow & posterior-inferior force on the upper humerus. The test is positive when pain is present. When the jerk test and Kim test are combined, there is 97% sensitivity in detecting a posterior labral tear.
Escobedo et al performed a retrospective study of 166 shoulder MRI arthrograms from patients who had presented with shoulder pain and/or instability. They selected out 20 competitive football players and 20 age-matched controls. 55% of the football players had at least one posterior labral tear, while only 10% of the matched non-football players had a posterior labral tear, which was statistically significant.
Acronyms are commonly used to describe injuries in sports medicine and may be tested with either the short or long name: SLAP=superior labrum anterior-posterior; ALPSA=anterior labral periosteal sleeve avulsion; PASTA=partial articular-sided supraspinatus tendon avulsion.
Acute posterior shoulder dislocations are less common than anterior dislocations, but more commonly missed
50% of traumatic posterior dislocations seen in the emergency department are undiagnosed
2% to 5% of all unstable shoulders
risk factors: bony abnormality, glenoid retroversion or hypoplasia is a less common cause of instability, ligamentous laxity
Pathophysiology
mechanism:
trauma (posterior dislocation) - 50% of cases that present for evaluation - usually dramatic presentation
microtrauma (posterior instability) - may lead to a labral tear, incomplete labral avulsion, or erosion of the posterior labrum. May lead to gradual stretching of capsule and patulous posterior capsule. Common in lineman, weight lifters, overhead athletes - usually insidious onset and presentation
seizures and electric shock: tetanic muscle contraction pulls the humeral head out - anterior instability and dislocations are still more common with seizures. However, posterior dislocations are unlikely to occur without significant trauma (ie. seizures)
biomechanical forces: flexed, adducted, and internally rotated arm is a high-risk position
Lesions Associated with Posterior Instability
Avulsion of posterior band of IGHL: Associated with acute subluxations
Posterior Bankart lesions: Characterized by detachment of posterior inferior capsulolabral complex
Reverse Hill-Sachs lesions :Associated with locked and difficult to reduce dislocations
Posterior labral cyst: Associated with chronic reverse Bankart lesion
Posterior glenoid rim fracture: Associated with chronic reverse Bankart lesion
Lesser tuberosity fracture: Associated with acute posterior dislocation
Large capsular pouch: Can see with MRI with contrast, often with chronic posterior instabiltiy
Glenohumeral anatomy
Primary stabilizers of the posterior shoulder
Posterior band of IGHL: primary restraint in internal rotation
subscapularis: primary dynamic restraint in external rotation, primary dynamic restraint against posterior subluxation
superior glenohumeral ligament and coracohumeral ligament : primary restraint to inferior translation of the adducted arm and to external rotation, primary static stabilizer to posterior subluxation with shoulder in flexion, adduction, and internal rotation
Static restraint: labrum deepens the glenoid by 50%
Classification
Acute versus chronic
Acute = trauma, seizure, electric shock with dramatic presentation
Chronic = microtrauma from repetition such as offense football lineman with insidious onset and presentation
Voluntary versus involuntary
Presentation
History: trauma or microtrauma with the arm in a flexed, adducted, and internally rotated position
chronic instability often presents with insidious onset, and vague symptoms (usually pain and not instability as opposed to anterior instability)
often in sporting or occupational activities that require repetitive pushing with the arm in forward flexed position foot ball lineman, weight lifters, etc
Symptoms
pain with flexion, adduction, and internal rotation of the arm
Physical exam
prominent posterior shoulder and coracoid for acute posterior dislocation
may be normal from chronic posterior instability from microtrauma
motion
limited external rotation for acute posterior dislocation
shoulder locked in an internally rotated position common in undiagnosed posterior dislocations
pain on flexion, adduction and internal rotation for posterior instability
provocative tests - performed in the setting of chronic posterior instability
posterior load & shift test: place patient supine with arm in neutral rotation with 40 to 60° abduction and forward flexion, load humeral head and apply anterior and posterior translating forces noting subluxation
Jerk test: place arm in 90° abduction, internal rotation, elbow bent, apply an axial force along axis of humerus and adduct the arm to a forward-flexed position, a ‘clunk’ is positive for posterior subluxation. 97% sensitive for posterior labral tear when combined with a Kim test
Kim test : performed by having the patient seated, arm at 90° abduction, followed by flexing the shoulder to 45 forward flexion while simultaneously applying axial load on the elbow & posterior-inferior force on the upper humerus.
test is positive when pain is present
posterior stress test: stabilize scapula and look for posterior translation with a posterior direct force
pain is elicited often, but this is not a specific finding
A 10-year-old boy presents to the emergency room after injuring his left knee while playing soccer. He localizes the pain to the distal femur, and is unable to bear weight on the affected leg. On physical exam the patient is tender to palpation only directly over the distal femoral physis. He has swelling about the distal thigh, without any signs of knee effusion. An AP and lateral radiograph of the affected knee are shown in Figures A and B (slight widening distal femoral physis, undisplaced) . An AP and lateral radiograph of the contralateral knee are shown in Figures C and D. What is the most appropriate treatment?
Hinged knee brace with early motion and weight bearing as tolerated
Cast immobilization with close clinical followup
Closed reduction and percutaneous pinning
Open reduction with pin fixation
Open reduction with plate fixation
Cast immobilization with close clinical followup
The clinical presentation, physical exam, and radiographs are consistent with a Salter-Harris Type I fracture of the distal femoral physis. The radiographs show subtle physeal widening, but no displacement. If there is no displacement following the injury, as in this case, then cast immobilization is acceptable treatment. However, these fractures are associated with a high incidence of deformity so close clinical followup is mandatory. If there is evidence of displacement with a SHI or SHII, then closed reduction percutaneous pinning would be indicated. Open reduction is reserved for SHIII and SHIV fractures, or fractures that can not be reduced.
Pediatric distal femoral physealfractures are most commonly Salter-Harris II fractures that resultfrom direct trauma in children with open physes. Treatment is usually closed reduction and percutaneous fixation followed by casting.
Occur in patients with open growth plates
must be considered in patients with open physes to avoid misdiagnosis with collateral ligament injury
Pathophysiology
Mechanism: often the result of direct trauma with some degree of rotation - most commonly a valgus-type force or a hyperextension force, most commonly Salter-Harris II fracture. The physis fails on the tension side and metaphysis fails on the compression side, creating a Thurston-Holland fragment
Prognosis
Physeal arrest
30-50% rate ofphyseal arrest thatoften leads togrowth disturbance and deformity - counsel parents of poor prognosis associated with this fracture pattern
an increased incidence of complications have been associated with: Salter-Harris classification type, fracture displacement, surgical hardware invading the physis
Anatomy
Osteology : formed from a single ossific nucleus that is present at birth and is the first epiphysis in the body to ossify
Muscles: both heads of the gastrocnemius and plantaris muscles originate just proximal to the physis, this leads to flexion of the distal fracture fragment when fracture line is distal to muscle insertion
Ligament : collateral ligaments attach distal to the physis at the level of the epiphysis - stress places tension on the collaterals which transfer force to the physis. ACL and PCL attach to epiphysis at the intercondylar notch and may be injured.
Blood Supply : femoral artery travels through the adductor canal medially above the metaphysis and courses in the popliteal space. The popliteal artery is directly posterior to distal femur and trifurcates at this level. Due to poor collateral circulation, popliteal artery injury may result in loss of lower limb viability
Physeal considerations of the knee
leg growth continues until: 16 yrs in boys, 14 yrs in girls
growth contribution: leg grows23 mm/year, with most of that coming from the knee (in contrast to upper extremity where most of growth is away from the elbow)
proximal femur - 3 mm / yr (1/8 in)
distal femur -9 mm / yr (3/8 in)
proximal tibia - 6 mm / yr (1/4 in)
distal tibia - 5 mm / yr (3/16 in)
Presentation
History of significant trauma
Symptoms: pain with inability to bend knee, unable to bear weight
Physical exam: pain and swelling, often in flexed position due to hamstring muscle spasm, tenderness along the physis in the presence of a knee effusion
may see varus or valgus knee instability on exam
swelling in the popliteal space may be a sign of vascular injury or disruption
Imaging
Radiographs - AP, lateral, oblique. Stress radiographsto look for physis opening if there was suspicion of physeal injury have fallen out of favor due to risk of physeal damage, patient discomfort, and possible need for sedation. MRI or ultrasound have replaced stress radiographs in this setting
Close follow up with repeat x-rays if inital radiographs not definitive and treating as a possible Salter I fracture
findings.
Physeal widening: normal 3-5mm
Direction of displacement suggestive of mechanism of injury
- Anterior displacement due to hyperextension
- Posterior displacement due to hyperflexion
- Medial displacement due to valgus
- Lateral displacement due to varus
MRI : indications - diagnostic modality of choice to confirm physeal fracture
Ultrasound : indications: can help confirm physeal fracture
CT : may be necessary for evaluation of intra-articular extension and to define fracture fragments to plan fixation
Angiography: occasionally necessary to evaluate for a vascular injury, (uncommon) - may be necessary in fractures with wide displacement and posterior spiked fragments
Treatment
Nonoperative - long leg casting: nondisplaced fractures
treated for 4-6 weeks. Close clinical follow up is mandatory
Operative
Closed reduction and percutaneous fixation followed by casting(majority of cases)
- displaced Salter-Harris I or II fractures
- displaced fractures successfully reduced with closed methods typically should still be secured with fixation as fracture pattern is unstable
- some Salter-Harris III or IV injuries if anatomic reduction is achieved
postoperativelyfollow closelyto monitor for deformity
ORIF
indications:
Salter-Harris III and IVwith weight-bearing articular involvement,
Irreducible SHI and SHII fractures (irreducible type II fractures are most often due to interposed periosteum on the tension side of fracture)
Techniques
Closed reduction and percutaneous fixation
- reduction: avoid multiple attempts at reduction, maneuver consists of 90% traction and 10% manipulation
- fixation : ifphysismust be crossed (SH I and SH II with small Thurston-Holland fragments), use smooth K-wires and remove 3-6 weeks after surgery. SH II fracture with large enough metaphyseal fragment should be fixed with lag screws across the metaphyseal segment
Open Reduction Internal Fixation
approach: incision over the displaced physis to remove interposed periosteum is necessary.
Fixationshould avoid the physis if possible
Complications
Limb length discrepancy or angular deformity(most common)- results from physeal disturbance
limb length inequality of >2 cm in one third of cases
Correlates with fracture pattern : 36% of SH 1 fractures 58% in SH 2fractures 49% in SH 3fractures 64% in SH 4 fractures
Minimize with: Anatomic physeal alignment (critical) and close follow up following nonoperative or operative
No treatment necessary when predicted final leg length discrepancy <2 cm and no significant angulation
Epiphysiodesis of the contralateral distal femur +/- proximal tibia: indicated fordiscrepancy between 2-6 cm
Physeal bridge excision: if physeal bar of <50% and
≥ 2 years or 2.5 cm of growth remaining
Septic arthritis
Intra-articular pins have a risk of septic arthritis
Popliteal artery injury and compartment syndrome
rare, most common with anterior displacement of epiphysis or a posterior spike at the fracture site
A 32-year-old man has difficulty walking 5 months after having an amputation as shown in Figure A (healed transmetatarsal amputation) . Gait analysis reveals a shortened stance phase and irregular toe off on the operative extremity. Which of the following would most likely improve his ambulation?
Solid ankle ankle-foot orthosis
Hip-knee-ankle-foot orthoses
University of California Biomechanics Laboratory (UCBL) insert
Full-length steel shank shoe modification
Full-length steel shank and rocker sole shoe modification
Full-length steel shank and rocker sole shoe modifications would most likely improve this patient’s gait. The steel shank replaces the lever arm length lost by toe amputation while the rocker bottom allows heel-toe weight transfer in the setting of decreased motion of the foot.
Rigid foot orthoses limit joint motion and stabilize flexible deformities. Semirigid orthoses allow dorsiflexion and/or plantar flexion of the ankle. Soft orthoses have the best shock-absorbing ability and are used to accommodate fixed deformities of the feet. Rocker soles help to transfer the body weight forward while walking, but they may destabilize the knee by transferring body weight forward too rapidly.
Mueller and Strube performed a cohort study of 30 patients that underwent a transmetatarsal amputation. They found that patients wearing full-length custom-made or a short shoe with a rigid rocker-bottom sole had significantly higher scores in the physical performance test (PPT) and faster walking speed than those wearing regular shoes with a toe filler.
Janisse and Janisse authored a Level 5 review discuss the importance of adding a rocker soles to rock the foot from heel strike to toe-off when a rigid extended shank is utilized in order to reduce bending stresses across the midfoot.
Figure A demonstrates a healed transmetatarsal amputation.
A 40-year-old male patient fell asleep at the wheel and was involved in a motor vehicle accident. At the emergency room, he presented with an ASIA C spinal cord injury. An AP radiograph is shown in Figure A. An axial CT scan at the C5 level is shown in Figure B (fracture line through lamina c5). Management of this injury should include:
Anterior cervical discectomy and fusion of C5-6
Corpectomy of C5 and instrumented fusion C5-6
Corpectomy of C5 and instrumented fusion C4-5
Posterior instrumented fusion of C4-6
Posterior instrumented fusion of C5-6
This patient has fracture separation of the lateral mass. This is best treated with posterior two-level fusion involving both the level above and the level below.
Posterior instrumented fusion of C4-6
Lateral mass-facet fractures are a result of hyperextension, lateral compression and rotation. Fracture separations of the lateral mass are not well seen on plain radiographs. On axial cut CT scans typically one fracture line is seen through the lamina, and the second through the pedicle. This compromises the stability of the inferior facet (of the vertebra above) and the superior facet (of the vertebra below), leaving the entire lateral mass free from the adjacent vertebral segment, yielding 2 levels of instability. Surgical fixation is indicated in the presence of neurologic injury or segmental instability and should involve instrumented fusion across both levels.
Lee et al. felt that it was unclear whether the management of these injuries should involve anterior, posterior or combined surgical approaches. In their series of 39 patients with 29 lateral mass fractures and 14 facet fractures, they found that nonsurgical treatment was usually unsuccessful, and single-level anterior fusion was adequate. However, for separation type fractures, they report two-level fusion is necessary.
Levine et al. examined 24 patients with this injury. They found that deformity (mean translation 5 mm and angulation 7 deg) occurred more commonly at the level below (80%) than the level above (20%). They opined that because fracture separations were a posterior injury, the preferred surgical approach would be posterior and advocated posterior lateral mass instrumentation.
Kotani et al. classified lateral mass fractures into 4 subtypes. Separation type fractures have 2 fracture lines involving the lamina and pedicle, leading to separation of the articular mass from the vertebral body. Comminution type fractures have multiple fracture lines, including lateral wedging and coronal plane deformity. Split type fractures have a vertical fracture line in the coronal plane, with separation of the lateral mass in the AP plane and upward migration of the superior articular process of the vertebra below. Traumatic spondylolysis fractures have bilateral horizontal fracture lines through the pars interarticularis.
Figure A shows lateralization of the left C5 lateral mass. Figure B shows floating left lateral mass (fracture lines through lamina and pedicle). Illustration A shows the Kotani classification of lateral mass fractures (A, separation; B, comminution; C, split; D, traumatic spondylolysis).
Incorrect Answers
Answer 1: Anterior stabilization is an option, but must involve both the levels above and below (C4-6) for this separation type fracture.
Answers 2,3: The vertebral body is not involved in this injury and corpectomy is unnecessary.
Answer 5: Posterior fusion of C5-6 alone is insufficient as this injury pattern results in 2 level instability (C4-5 and C5-6).
Cervical Lateral Mass Fracture Separation: Fracture separations of the lateral mass-facet are uncommon Injuries characterized by:
- high degree of instability
- neurological deficit
- affect 2 levels (2 adjacent motion segments) because of involvement of the superior facet and inferior facet on either side of the fractured articular mass
Mean age 35 yrs (20-70yrs)
location : C6 > C5 > C7 > C4 > C3
Pathophysiology
Mechanism of injury : traffic accident, falls, heavy object landing on head. Hyperextension, lateral compression and rotation of the cervical spine
Associated conditions
- anterior translation (listhesis): fractured vertebrae (77%), superior adjacent vertebrae (24%), inferior adjacent vertebrae (10%): coronal translation (33%)
- vertebral body collapse (33%): lower in Type A
Kotani Classification - Separation fracture subtypes
Type A - Separation fracture
2 fracture lines of unilateral lamina and pedicle
Type B - Comminution type
Multiple fracture lines with lateral wedging in coronal plane
Type C - Split type
Vertical fracture line in the coronal plane, with invagination of the superior articular process of the caudal vertebra
Type D - Traumatic spondylolysis
Bilateral horizontal fracture lines of the pars interarticularis, leading to separation of the anterior-posterior spinal elements
A 38-year-old male complains of progressive right groin pain over the past year. Initially, the pain occurred only with weight-bearing, but is now beginning to bother him while sitting and has awoken him from sleep on a few occasions. A radiograph demonstrates a mixed lucent and sclerotic lesion within the femoral head just below the articular surface. An MRI is shown in figure A. A histology slide from an open biopsy is provided in figure B (chondrocytes with a clear, vacuolated cytoplasm). Which of the following is the most likely diagnosis?
Lymphoma Clear cell chondrosarcoma Chondroblastoma Giant cell tumor Osteosarcoma
The clinical presentation, MRI, and histology are consistent with clear cell chondrosarcoma. The histology demonstrates chondrocytes with a clear, vacuolated cytoplasm in a chondroid matrix, which is characteristic of clear cell chondrosarcoma. In addition, the lesion is in the epiphysis which is also typical of clear cell chondrosarcoma. Radiographs of clear cell chondrosarcoma usually demonstrate mixed lucent and sclerotic lesions with a paucity of matrix densities or calcifications.
“Popcorn calcifications” typically seen in classic chondrosarcoma are not prevalent on radiographs of clear cell chondrosarcoma. Chondroblastoma and giant cell tumors are two other epiphyseal lesions that must be differentiated from clear cell chondrosarcoma. Chondroblastomas are predominantly found in teenagers and young adults. Giant cell tumors will demonstrate classic giant cells on pathology.
Malignant primary bone tumor composed of chondrocytes with variable degrees of malignancy
Primary chondrosarcoma which includes low-grade, high-grade,dedifferentiated chondrosarcoma
clear cell chondrosarcoma, mesenchymal chondrosarcoma
Secondary chondrosarcoma: arises from benign cartilage lesions including:
- osteochondroma(<1% risk of malignant transfomation)
- multiple hereditary exostosis(1-10% risk of malignant transformation)
- enchondromas(1% risk of malignant transformation)
- Ollier’s disease(25-40% risk of malignant transformation)
- Maffucci’s(100% risk of malignant transformation)
Chondrosarcomas are found inolder patients (40-75 yrs)
there is a slight male predominance. Most common locations include thepelvis, proximal femur, distal femur, scapula- tumor location is important for diagnosis as the same histology may be diagnosed as benign in the hand but malignant if located in the long bones
Prognosis: histologic grade correlates with survival
Grade I : 90% 5-year survival - grade 1 chondrosarcomas are generally slow growing
Grade II : 60-70%5-yearsurvival
Grade III : 30-50%5-yearsurvival
De-differentiated chondrosarcoma <10%5-yearsurvival
Poor prognostic variables
- Axial and proximal extremity lesions have a more aggressive course
- Pelvic location
- Advanced patient age, inadequate surgical margins
Increased telomerase activity - in chondrosarcoma, as determined by reverse transcriptase-polymerase chain reaction (RT-PCR), has been shown to directly correlate withthe rate of recurrence
Which classification system for cervical myelopathy focuses exclusively on lower extremity function?
Nurick Japanese Orthopaedic Association Modified Japanese Orthopaedic Association Ranawat Oswestry
The Nurick Classification system is a classification system for cervical myelopathy that focuses on the ambulatory status of the patient.
The different Grades of the Nurick classification system include: (GRADE 0) Signs or symptoms of root involvement but without evidence of spinal cord disease. (GRADE 1) Signs of spinal cord disease but no difficulty in walking. (GRADE 2) Slight difficulty in walking that did not prevent full-time employment. (GRADE 3) Difficulty in walking that prevented full-time employment or the ability to perform all housework but that was not severe enough to require someone else’s help to walk. (GRADE 4) Able to walk with someone else’s help or the aid of a frame. (GRADE 5) Chair bound or bedridden.
Hirabayashi et al present a Level 4 study of 53 cases who underwent surgical decompression for patients with OPLL and concomitant myelopathy or radiculopathy. They reported a recovery rate of approximately 70%.
Edwards et al present a Level 5 review of cervical myelopathy. They report that anterior decompression and fusion procedures including 1 or 2 levels have predictable results, but ACDFs involving 3 or more levels are associated with increased morbidity.
Incorrect Answers:
Answer 2: The Japanese Orthopaedic Association Classification system includes chopstick function.
Answer 3: The Modified Japanese Orthopaedic Association Classification system includes upper extremity dexterity tasks such as buttoning of a shirt.
Answer 4: The Ranawat Classiciation takes into account upper extremity, and well as lower extremity, symptoms.
Answer 5: The Oswestry Disability Index is a classification for back pain, not cervical myelopathy.
Cervical myelopathy is a common degenerative condition caused by compression on the spinal cord that is characterized by clumsiness in hands and gait imbalance.
treatment is typically operative as the condition is progressive.
Pathophysiology
Etiology:
- Degenerative cervical spondylosis (CSM) - most common cause of cervical myelopathy. Compression usually caused by anterior degenerative changes (osteophytes, discosteophyte complex). Degenerative spondylolisthesis and hypertrophy of ligamentum flavum may contribute
- Congenital stenosis: symptoms usually begin when congenital narrowing combined with spondylotic degenerative changes in older patients
- OPLL (Ossification Posterior Longitudinal Ligament)
- tumor
- epidural abscess
- trauma
- cervical kyphosis
Neurologic injury - mechanism can be direct cord compression and/or ischemic injury secondary to compression of anterior spinal artery
Associated conditions
lumbar spinal stenosis: tandem stenosis occurs in lumbar and cervical spine in ~20% of patients
Prognosis
Tends to be slowly progressive and rarely improves with nonoperative modalities - progression characterized by steplike deterioration with periods of stable symptoms
prognosis.
Early recognition and treatment prior to spinal cord damage is critical for good clinical outcomes
Classification of Myelopathy
Nurick Classification
Grade 0 Root symptoms only or normal
Grade 1 Signs of cord compression; normal gait
Grade 2 Gait difficulties but fully employed
Grade 3 Gait difficulties prevent employment, walks unassisted
Grade 4 Unable to walk without assistance
Grade 5 Wheelchair or bedbound
Based on gait and ambulatory function
Ranawat Classification
Class I Pain, no neurologic deficit
Class II Subjective weakness, hyperreflexia, dyssthesias
Class IIIA Objective weakness, long tract signs, ambulatory
Class IIIB Objective weakness, long tract signs, non-ambulatory
Japanese Orthopaedic Association Classification
A point scoring system (17 total) based on function in the following categories
upper extremity motor function
lower extremity motor function
sensory function
bladder function
Symptoms
- neck pain and stiffness: axial neck pain (often times absent), occipital headache common
- extremity paresthesias: diffuse nondermatomal numbness and tingling
- weakness and clumsiness: weakness and decreased manual dexterity (dropping object, difficulty manipulating fine objects)
- gait instability : patient feels “unstable” on feet, weakness walking up and down stairs, gait changes are most important clinical predictor
- urinary retention: rare and only appear late in disease progression (not very useful in diagnosis due to high prevalence of urinary conditions in this patient population)
Physical exam
Motor
- weakness: usually difficult to detect on physical exam - lower extremity weakness is a more concerning finding
- finger escape sign: when patient holds fingers extended and adducted, the small finger spontaneously abducts due to weakness of intrinsic muscle
- grip and release test: normally a patient can make a fist and release 20 times in 10 seconds. myelopathic patients may struggle to do this
Sensory
- proprioception dysfunction: due to dorsal column involvement - occurs in advanced disease & associated with a poor prognosis
- decreased pain sensation: pinprick testing should be done to look for global decrease in sensation or dermatomal changes. Due to involvement of lateral spinothalamic tract
(vibratory changes are usually only found in severe case of long-standing myelopathy)
Upper motor neuron signs (spasticity)
- hyperreflexia: may be absent when there is concomitant peripheral nerve disease (cervical or lumbar nerve root compression, spinal stenosis, diabetes)
- inverted radial reflex: tapping distal brachioradialis tendon produces ipsilateral finger flexion
- Hoffmann’s sign : snapping patients distal phalanx of middle finger leads to spontaneous flexion of other fingers
- Sustained clonus post > three beats defined as sustained clonus. Sustained clonus has poor sensitivity (~13%) but high specificity (~100%) for cervical myelopathy
- Babinski test post: considered positive with extension of great toe
Gait and balance
- toe-to-heel walk: patient has difficulty performing
- Romberg test : patient stands with arms held forward and eyes closed - loss of balance consistent with posterior column dysfunction
Provocative tests
- Lhermitte Sign: test is positive when extreme cervical flexion leads to electric shock-like sensations that radiate down the spine and into the extremities
An 8-year-old boy’s parents are concerned about the appearance of the child’s middle finger. The child denies pain and his digital neurovascular status is normal. A clinical photograph and radiograph are provided in figures A and B (macrodactyly middle finger). For children with this condition, which of the following is the best intervention to achieve a finger that is proportional to the rest of the hand?
Epiphysiodesis now
Epiphysiodesis when the finger reaches adult length of the father
Compression wrapping until proportional size is achieved
Resection of hypertrophic nerves
Osteotomy at skeletal maturity.
Epiphysiodesis when the finger reaches adult length of the father
Clinical photograph and radiographs demonstrate macrodactyly of the middle finger, a rare congenital malformation enlarging all structures of the digit.
Ishida et al reviews 23 cases of surgically treated macrodactyly finding favorable results with epiphysiodesis/epiphysiodectomy while resection of hypertrophic nerves was unsuccessful in preventing overgrowth. The epiphysiodesis is performed once the finger reaches the length of the same sex parent, using their digit as a template for final growth.
Nonhereditary congenital digit enlargement: very rare
90% are unilateral
70% involves more than one digit
index involved most frequently - in order of decreasing frequency, the long finger, thumb, ring, and small are also involved
Can involve digits of the hand or foot
Risk factors and aetiology unknown, no genetic correlations known to date
Affected digits correspond with neurologic innervation- the median nerve being the most common
Associated conditions - Lipfibromatous hamartoma of the median nerve is the adult homolog Has been associated with: Proteus syndrome Banayan-Riley-Ruvalcabe's disease Maffucci syndrome Ollier’s disease Milroy’s disease
Prognosis
if static, asymmetry does not worsen
if progressive, asymmetry worsens with time
Classification
Functional Classification
Static: Present at birth and growth is linear with other digits
Progressive: Not as noticable at birth but shows disproportionate growth over time
Symptoms: pain, inability to use digits, complaints of cosmetic issues
Physical exam
inspection & palpation: thick, fibrofatty tissue involving enlarged digits
ROM & instability: often limited ROM due to soft tissue constraints
Imaging
Biplanar hand radiographs: enlarged phalanges to involved digits - may see malalignment of joints or angled phalanges
CT, MRI: not typically needed
Angiography: only needed if used for surgical planning
Treatment
Nonoperative: observation in mild cases
Operative
Epiphysiodesis - indications: single digit
perform once digit reaches adult length of same sex parent
most common approach
Postoperative care: soft tissue care, early ROM
Osteotomies and shortening procedures - indications: thumb involvement, multiple digit involvement, severe deformity
Postoperative care: local soft tissue care, early ROM
Amputations if severe involvement of digit/ non-reconstructable digit
Complications
Digital stiffness
Chronic digital pain or edema
A 25-year-old healthy, active male undergoes lateral closing wedge high tibial osteotomy. Which of the following complaints is most commonly associated with this procedure?
Joint laxity Infection Anterior knee pain Quadricep weakness Limb lengthening
The most common complaint associated with lateral closing wedge high tibial osteotomy is anterior knee pain.
Lateral closing wedge high tibial osteotomies are commonly associated with anterior knee pain due to the high incidence of patella baja post-operatively. Patella baja is characterized by the lowering of the patella relative to its normal position, which is typically measured using the Insall-Salvati ratio of < 0.8 (Normal ratio = 0.8 - 1.2). The most common symptoms associated with patella baja include anterior impingement, knee pain, and knee stiffness.
Scuderi at al. evaluated the effect of proximal tibial osteotomy on patellar height in 66 patients. They found that patellar height decreased by 89%, as measured by the Insall-Salvati index, and 76.3%, as measured by the Blackburne-Peel index, post-operatively.
Wright et al. reviewed the complications associated lateral closing wedge and medial opening wedge high tibial osteotomy. They reported a 64% incidence of patella baja following these high tibial osteotomy techniques, with associated complaints of anterior knee pain.
Illustration A shows an X-Ray of patella baja after closed wedge high tibial osteotomy. The Insall-Salvati ratio is determined by measuring the ratio of patella tendon length (TL) to the length of the patella bone (PL) with the knee flexed at 30 deg.
Incorrect Answers:
Answer 1: Knee stiffness, not laxity, is more common after high tibial osteotomy.
Answer 2: The incidence of deep infection is roughly 0 - 4%.
Answer 4: Quadriceps weakness typically exists prior to high tibial osteotomy.
Answer 5: Leg shortening is more likely associated with lateral closing wedge high tibial osteotomy.
Patella baja is characterized by lowering of the patella relative to its normal position- may be congenital or acquired
Common causes include
- proximal tibial osteotomy - patella baja is the most common complication seen following proximal tibial opening-wedge osteotomy. May be caused by shortening of the patellar tendon during tibial osteotomy or from scarring of the patellar tendon post-operatively
- tibial tubercle slide or transfer
- trauma to the proximal tibia
- technical error during primary total knee replacement (joint line elevation)
- ACL reconstruction
Associated conditions
total knee arthroplasty: patella infera is an important consideration when performing total knee arthroplasty
improper technique may cause patella baja
special techniques must be utilized when performing TKA in patients with patella baja from congenital or acquired (tibial osteotomy, prior TKA) causes
A 22 year-old college cross-country runner developed hip and groin pain that initially started while running, but is now painful when walking across campus. Radiographs show no evidence of a stress fracture, an alpha angle of 45 degrees, and a lateral center edge angle of 30 degrees. An MRI shows focal, intense marrow edema in the superior-lateral femoral neck. What is the most appropriate treatment?
Operative treatment with percutaneous screw placement
Hip arthroscopy with cam resection
Hip arthroscopy to treat both cam and pincer impingment
Non-operative treatment with NSAIDs and reduction in mileage
Non-operative treatment with partial weight-bearing
The history and MRI findings are consistent with a tension-side femoral neck stress fracture for which the most conservative treatment is percutaneous screw placement.
Groin pain in an athlete is a nonspecific complaint with a large differential diagnosis, including adductor strain, iliopsoas tendinitis, FAI, labral tears, and stress fractures. Treatment of non-displaced tension-side fractures is still somewhat controversial as there are no prospective studies comparing nonoperative and operative treatment. Most authors, mindful of the potential for serious complications should this fracture displace, view tension-side fractures with a great deal of respect and recommend either extremely cautious conservative treatment or internal fixation.
In the review by Shin & Gillingham, they recommend treating tension-side injuries aggressively with cannulated screws as soon as the diagnosis is clear to avoid the potentially catastrophic sequelae of displacement, prevent the harmful effects of bedrest, and allow early weight bearing with crutches.
The article by Hajek & Noble from 1982 was one of the early case series describing femoral neck stress fractures in distance runners.
Illustration A shows a fluid sensitive MRI hip coronal view with linear marrow edema extending from the lateral head-neck junction across most of the femoral neck. This is consistent with a non-displaced stress fracture.
Incorrect Answers:
Answers 2,3: These are procedures for femoral-acetabular impingement, not for stress fractures. The measurements described in the stem are within the normal range.
Answers 4,5: Non-operative treatment is not usually recommended, but successful reports in the literature have used strict non-weight-bearing with 3+ weeks of bedrest, followed by prolonged crutch use.
Fracture of the femoral neck secondary to repetitive loading of bone - common in runners. Repetitive loading causes microscopic fractures in the femoral neck - crack “initiation.” Then continued repetitive loading does not allow for healing response and stress fracture occurs - crack “propagation”
two types
- compression side (inferior-medial neck)
- tension side (superior-lateral neck)
Associated conditions
“female athlete triad”
amenorrhea, eating disorder, and osteoporosis
must be considered in any female athlete with stress fracture
Prognosis: dependent upon patient compliance
Muscle insertions around femoral neck iliopsoas quadratus femoris gluteus medius piriformis
Blood supply to femoral neck
provided by two branches of the femoral artery including
medial femoral circumflex artery
lateral femoral circumflex artery
Biomechanics of femoral neck
compression side : inferior medial neck with weight bearing
tension side: superior lateral neck with weight bearing
Treatment
Nonoperative: non-weight bearing, crutches and activity restriction
indications: compression side stress fractures with fatigue line <50% femoral neck width
Operative
ORIF with percutaneous screw fixation
indications:
- tension side stress fractures
- compression side stress fractures with fatigue line >50% femoral neck width
- progression of compression side stress fractures
Technique: Use three 6.5mm or 7.0mm cannulated screws
Postoperative weightbearing as tolerated
Complications
Fracture progression/completion
if fracture is unrecognized and the athlete continues to train
fracture completion is associated with severe, disabling complications
precludes return to prior activity level (elite athletes will not be able to return to prior level following displaced fracture)
Varus settling
AVN
Nonunion
Refracture
An 18-year-old girl presents with a deformity of the left leg that limits her ability to play basketball and volleyball. She reports pain along the lateral joint line with vigorous activity. A clinical image of the left leg in the supine position is shown in Figure A (genu valgum). A standing alignment radiograph is shown in Figure B with the mechanical lateral distal femoral angle measured at 73° (mLDFA 88°, range 85°-90°), an mechanical medial proximal tibial angle of 87° (mMPTA 87°, range 85°-90°), and a tibial femoral angle of 25°(range 5°-10°). Which of the following is the most appropriate surgical treatment?
Lateral closing wedge proximal femoral osteotomy with medial opening wedge tibial osteotomy Lateral closing wedge tibial osteotomy Medial opening wedge femoral osteotomy Medial closing wedge tibial osteotomy Medial closing wedge femoral osteotomy
Medial closing wedge femoral osteotomy
Genu valgum in the skeletally mature individual is most appropriately treated with a varus-producing distal femoral osteotomy. If the deformity were to be addressed with a medial closing wedge tibial osteotomy an oblique joint line would be maintained. Varus and valgus can be determined by measuring the lateral distal femoral angle (normal=88 deg), medial proximal tibial angle (normal=87 deg), or the tibiofemoral angle (normal=5-10 degrees). Figure B shows that the the lateral distal femoral angle is less than 87 degrees and thus the femur should be addressed with the deformity correction.
Tetsworth and Paley present a Level 4 review of 28 limbs that underwent external fixation for deformity correction. They measured mechanical axis deviation (MAD) and mechanical tibiofemoral angle (mTFA) pre and postoperatively in these patients and found external fixation to be a reliable method of deformity correction.
Paley presents a Level 5 review discussing the spectrum of lower extremity angular and rotational deformities and their treatments. He cites that the work of Gavril Abramovich Ilizarov in the field of external fixation has been instrumental in advancing the field of deformity correction and the magnitude of the deformity is no longer an obstacle with gradual correction of bone or joint deformities.
Genu valgum is a normal physiologic process in children
therefore it is critical to differentiate between a physiologic and pathologic process
Distal femur is the most common location of primary pathologic genu valgum but can arise from tibia
Etiologies
- Bilateral genu valgum: physiologic, renal osteodystrophy (renal rickets), skeletal dysplasia (Morquio syndrome, spondyloepiphyseal dysplasia, chondroctodermal dysplasia)
- Unilateral genu valgum: physeal injury (from trauma, infection, or vascular insult), proximal metaphyseal tibia fracture, benign tumors (fibrous dysplasia, osteochondromas, Ollier’s disease)
Prognosis
The threshold of deformity that leads to future degenerative changes is unknown
Deformity after a proximal metaphyseal tibia fracture (Cozen) should be observed, as it almost always remodels
Anatomy
Normal physiologic process of genu valgum
- between 3-4 years of age children have up to 20 degrees of genu valgum
- genu valgum rarely worsens after age 7
- after age 7 valgus should not be worse than 12 degrees of genu valgum
- after age 7 the intermalleolar distance should be <8 cm
Treatment
Nonoperative: observation is first line of treatment - genu valgum <15 degrees in a child <6 years of age
bracing is rarely used - ineffective in pathologic genu valgum and unnecessary in physiologic genu valgum
Operative:
Hemiepiphysiodesis or physeal tethering (staples, screws, or plate/screws) of medial side
indications:
> 15-20° of valgus in a patient <10 years of age
if line drawn from center of femoral head to center of ankle falls in lateral quadrant of tibial plateau in patient > 10 yrs of age
technique
- to avoid physeal injury place them extraperiosteally
- to avoid overcorrection follow patients often
- growth begins within 24 months after removal of the tether
Distal femoral varus osteotomy
indications
insufficient remaining growth for hemiepiphysiodesis
Complications
peroneal nerve injury - perform a peroneal nerve release prior to surgery
Gradually correct the deformity- utilize a closing wedge technique
Which of the following investigative studies is most useful in the definitive diagnosis of Amyotrophic Lateral Sclerosis (ALS)?
Genetic testing MRI brain and spinal cord Muscle biopsy Serum protein electrophoresis and immunoelectrophoresis Electrodiagnostic studies
The diagnosis of ALS requires a period of clinical observation to document the progressive loss of upper and lower motor function. Electrodiagnositic studies are required to make a definitive diagnosis.
Amyotrophic lateral sclerosis (ALS) is the most common degenerative disease of the motor neuron system. Also referred to as Lou Gehrig’s disease.
A progressive motor neuron disorder characterized by involvement of anterior horn cells of spinal cord.
Nerve conduction studies and needle electromyography (EMG) are useful for confirming the diagnosis of ALS and for excluding peripheral conditions that resemble ALS. Hallmark findings in the electrodiagnosis of ALS are abnormal motor nerve conduction studies, with normal sensory nerve conduction studies.
EMG / NCS - shows denervation + reinnervation
widespread decreased amplitude of CMAP and slowed motor conduction velocity
- denervation (fibrillations and positive waves) + decreased recruitment in ≥ 3 extremities
- reinnervation
- abnormal spontaneous fibrillation & fasciculation potentials
- normal sensory studies (SNAP, sensory nerve action potentials)
UMN signs are mild weakness, spasticity, and abnormally brisk reflexes; LMN signs are progressive weakness, wasting, and loss of reflexes and muscle tone.
Brooks et al. developed a diagnostic algorithm for the diagnosis of ALS. The algorithm is based on the degree of certainty of diagnosis, which is increased by the number of body segments that demonstrate upper motor neuron (UMN) and lower motor neuron (LMN) abnormalities. Clinical and electrophysiologic findings in 3 or more body segments is definitive of the diagnosis.
Incorrect Answers:
Answer 1: Genetic testing may be performed to identify genetic defects in some familial types of ALS. However to date, there is no single test that is definitive for diagnosing ALS.
Answer 2: MRI brain and spinal card are used to rule out structural lesions and neurologic conditions that have a slimilar clinical presentation as ALS.
Answer 3: Muscle biopsy is not required in the diagnosis of ALS
Answer 4: Serum protein electrophoresis and immunoelectrophoresis will not provide a definitive diagnosis for ALS. They are used to rule out the differential diagnoses of ALS.
Differentials
Peripheral compressive neuropathy: hyperreflexia and other UMN signs (Babinski, Hoffman) are present in ALS (which can present in a single extremity mimicking cubital/carpal tunnel syndrome), but absent in peripheral neuropathy
ALS has normal sensory studies on EMG/NCS
Treatment
Nonoperative - currently no cure or effective treatment
Goals of treatment: provide supportive care, prevent progression, maintain independent patient function and comfort
riluzole
Modest benefits only, prolongs life by 2-3 months
Blocks tetrodotoxin-sensitive sodium channels associated with damaged neurons - delays onset of ventilator-dependence and may prolong survival
34-year-old active duty military officer has lateral right ankle pain with running during physical training that is worsening over the past 6 months. He recalls sustaining an ankle sprain 3 years ago that resolved with physical therapy. On examination the skin is intact and the talar drawer test is normal. He has pain anterolaterally with end-arc passive dorsiflexion and no pain posteriorly with passive plantarflexion. He has no tenderness on palpation at the distal fibula, anterior talofibular ligament, calcaneofibular ligament. An axial MR arthrogram of the ankle is shown in Figure A (anterolateral soft tissue). What is the next most appropriate step in management following recalcitrant pain despite conservative management?
Ankle arthropscopy with synovial debridement
Open Brostrom ligament repair with Gould modification
Chrisman-Snook tendon transfer
Syndesmosis reduction and screw fixation
Ankle arthroscopy with loose body removal
The patient has a clinical presentation consistent with anterolateral ankle soft tissue impingement. Ankle arthroscopy with synovial debridement is indicated in cases recalcitrant to conservative management.
Ferkel et al. present a Level 4 study evaluating 24 patients who had an arthroscopic diagnosis of anterolateral soft tissue impingement. They then reviewed the MR images of the patients and found a 78.9% accuracy in diagnosing anterolateral soft tissue impingement.
Ferkel et al. also conducted a Level 4 study of 31 patients that underwent a partial synovectomy with debridement of scar tissue from the lateral gutter. They found that most patients had a good to excellent result at 2 year follow-up.
Figure A demonstrates anterolateral soft tissue as shown by the arrow. The common region for anterolateral ankle soft tissue impingement between the ATFL and anterior inferior tibiofibular ligaments.
Incorrect Answers:
Answer 2: A Brostrom repair and Gould modification is an anatomic shortening and reinsertion of the ATFL and CFL reinforced with the inferior extensor retinaculum and distal fibular periosteum
Answer 3: Chrisman-Snook procedure is a nonanatomic reconstruction using a split peroneus brevis tendon to reinforce the anterior talofibular and calcaneofibular ligaments.
Answer 4: Syndesmosis reduction and screw fixation is indicated in syndesmosis diastasis.
Answer 5: There is no loose body evident in this patient’s presentation.
Tibiotalar Impingement
Most often caused by osteophyte impingement in anterior tibiotalar joint
Can also be caused by excessive anterolateral soft tissues or posterior soft tissue or osseous abnormalities
Common in athletes who play on turf or on grass including rugby, football, dancers, soccer
Mechanism: repetitive overuse injuries, trauma, degenerative sequelae
Presentation
Symptoms: pain in anterior ankle
Physical exam: pain with forced dorsiflexion, limited dorsiflexion, soft tissue swelling and effusion may be evident, subtalar joint is pain free
Imaging
Radiographs: AP, lateral, and oblique - spurs seen in anterior distal tibia or dorsal aspect of the talus
oblique views are beneficial in revealing anteromedial talar spurs
CT: delineates extent of bony osteophytes
MRI: shows spurring and fluid in joint
Treatment
Nonoperative: therapy, lifestyle modifications, NSAIDS - first line of treatment
corticosteroid injections - help with soft tissue impingement and synovitis-related pain
Operative
arthroscopic excision
indications: nonoperative modalities fail
Techniques
Arthroscopic excision - supine position with external traction device and leg over a padded bump. Use knife to only cut the skin and use hemostat to spread to avoid neurovascular injury while making portals. Ensure adequate field of view prior to burring or shaving anterior distal tibia to avoid iatrogenic dorsal NV bundle injury
Complications
Superficial peroneal nerve injury during anterolateral portal creation
Saphenous vein injury during anteromedial portal creation
Dorsal neurovascular bundle injury during tibiotalar spur removal
Which of the following patients would benefit most from a glenohumeral arthrodesis?
A 74-year-old man with AVN of the proximal humerus
A 24-year-old woman with recurrent instability after an arthroscopic procedure for multi-directional instability
A 30-year-old laborer with a paralysis of the deltoid and rotator cuff muscles
A 70-year-old patient with removal of an infected primary total shoulder arthroplasty
A 20-year-old man patient with cleidocranial dysostosis
A 30-year-old laborer with a paralysis of the deltoid and rotator cuff muscles
A glenohumeral arthrodesis would be most beneficial to a patient with a flail shoulder but intact elbow and hand function. This is especially important for a laborer. It will allow the patient to have a stable base upon which to be able to use the hand. The position of arthrodesis is described in the review topic, and the goal is to position the shoulder so that the hand can reach the patient’s mouth for feeding and groin for hygiene. The goal of shoulder arthrodesis is to provide a stable base for the upper extremity optimizing hand and elbow function
it remains an important treatment option in appropriately selected patients
According to Clare et al. the indications include posttraumatic paralysis of the deltoid muscle and rotator cuff, brachial plexus injuries, chronic infection, failed revision arthroplasty, severe refractory instability, and bone deficiency following tumor resection.
The review by Safran & Iannotti noted marked improvements in pain and function in appropriately selected patients. Complications include nonunion, malposition, pain associated with prominent hardware, and periarticular fractures.
Incorrect Responses:
- Hemiarthoplasty may be better for a younger patient with AVN.
- Continued therapy and revision arthroscopic or open stabilization would be better choices.
- Once the infection is treated, a second-stage re-implantation of a TSA, hemiarthroplasty or reverse TSA would be a better option.
- Patients with cleidocranial dysostosis may benefit from scapulo-thoracic arthrodesis, not glenohumeral.
Indications
- stabilization of paralytic disorders
brachial plexus palsy
- irreparable deltoid and rotator cuff deficiency with arthropathy
- salvage of a failed total shoulder arthroplasty
- reconstruction after tumor resection
- painful ankylosis after chronic infection
- recurrent shoulder instability which has failed previous repair attempts
- paralytic disorders in infancy
Contraindications
- ipsilateral elbow arthrodesis
- contralateral shoulder arthrodesis
- lack of functional scapulothoracic motion
- trapezius, levator scapulae, or serratus anterior paralysis
- Charcot arthropathy during acute inflammatory stage (Eichenholtz 0-2)
- elderly patients
- progressive neurologic disease
Anatomy
Glenohumeral articulation: a relatively small amount of surface area exists allowing for predictable fusion. To increase the available fusion area, decortication of both the glenohumeral articular surface and the articulation between the humeral head and the undersurface of the acromion is performed. Only the glenoid fossa and base of the coracoid provide sufficient strength for fixation
Symptoms: specific to the underlying condition necessitating arthrodesis, symptomatic dysfunction of the glenohumeral joint
A 8-year-old boy presents with knee pain and an effusion. Biopsy and staging studies show a distal femoral osteosarcoma with contamination of the knee joint. Which of the following treatment options will provide this child with the best chance of local control and the highest level of function?
Through knee amputation
Above knee amputation
Rotationplasty
Extra-articular resection, endoprosthetic reconstruction, and free flap coverage
Extra-articular resection, allograft prosthetic composite, and free flap coverage
Rotationplasty
While intra-articular extension of osteosarcoma is a rare presentation, understanding of the various treatment options available is crucial for the treating surgeon. Often joint contamination occurs secondary to a previous surgical procedure when a malignant neoplasm isn’t considered in the differential diagnosis. While limb salvage may be possible with joint contamination, optimization of function and limitation of the number of future surgical procedures needs to be considered. Rotationplasty offers the treating surgeon the ability to perform a safe and negative margin resection while at the same time maximizing patient function and limiting the number of future re-operations.
Damron et al review the various joint-related tumors which mimic sports-related injuries in their instructional course lecture. They suggest that appropriate preoperative imaging and consideration of malignant conditions in the differential diagnosis of any suspicious condition will increase the chances that the treating surgeon correctly identifies the neoplasm and avoids a procedure which compromises limb-salvage.
Quan et al retrospectively reviewed 27 cases of peri-articular osteosarcoma, specifically looking for intra-articular involvement. They suggest that intra-articular involvement of osteosarcoma is very rare as articular cartilage serves as a good boundary to tumor expansion. Joint contamination from an incorrect surgical procedure is a much more likely cause of intra-articular extension.
Incorrect answers
1) Through knee amputation - with joint contamination, a through knee amputation would not provide an appropriate negative margin resection and therefore contaminate the distal resection margin with tumor
2) Above knee amputation - while this would provide a safe resection level, the level of function would not be as great as with a rotationplasty
4 & 5) Both of these options would be available for limb salvage, however they would not result in as high a level of function as a rotationplasty. In addition these procedures would likely require many subsequent operations given the higher rates of complication and the need for future prosthetic lengthening/exchange.
Intramedullary osteosarcoma is the most common primary sarcoma of bone
- the most common malignancy of bone is metastatic disease
- the most common primary malignancy of bone is myeloma
Bimodal distribution of occurrence
- majority occur in the second decade of life
- second peak in occurrence is in elderly patients with Paget’s disease
most common site is the distal femur and proximal tibia
other common sites include proximal humerus, proximal femur, and pelvis
Malignancy: most commonly diagnosed as Stage IIB (high grade, extra-compartmental, no metastases)
10-20% of patients present with pulmonary metastases (obtain CT of chest) - lung is most common site of metastasis
bone is second most common site
Genetics
patients who carry the Retinoblastoma tumor suppressor gene (Rb) are predisposed to osteosarcoma
risk increased in Rothmund Thomson syndrome - AR inheritance, mutations in RECQL4 gene, chr 8q24.3
sun-sensitive facial poikiloderma rash (pigmentation, thinned skin, prominent blood vessels), absent eyelashes, eyebrows, hair, juvenile cataracts, teeth abnormalities, osteosarcoma, fibrosarcoma, gastric adenocarcinoma, cutaneous BCC and SCC
Prognosis
76% long-term survival with modern treatment
poor prognostic factors include: advanced stage of disease (most predictive of survival), response to chemotherapy (as judged by percent tumor necrosis of resected specimen), tumor site and size, expression of P-glycoprotein, high serum alkaline phosphatase, high lactic dehydrogenase, vascular involvement, surgical margins, type of chemotherapy regimen
Symptoms: rapidly progressive pain, fever, and swelling
Physical exam: may feel mass on examination
Imaging
Radiographs: characteristic blastic and destructive lesion - sun-burst or hair on end pattern of matrix mineralization, periosteal reaction (Codman’s triangle), large soft tissue mass with maintenance of bone cortices
MRI: must include entire involved bone to determine
soft tissue involvement, neurovascular involvement, presence skip metastases - if skip metastases are found, this is equivalent to metastatic (stage III) disease
Bone scan: very hot in osteosarcoma. Useful to evaluate extent of local disease and presence of bone metastases
CT: chest CT is required at presentation to evaluate for pulmonary metastases
Labs
Elevated alkaline phosphatase - may be 2-3 times normal value
Histology: tumor cells show significant atypia, and produce “lacey” osteoid. Stroma cells show malignant characteristics with atypia, high nuclear to cytoplasmic ratio, and abnormal mitotic figures. May have mixed histology with different combinations of chondroblastic,osteoblastic, or fibroblastic looking cells depends on the subtype of osteosarcoma. Giant cells may be present in giant cell rich osteosarcoma - often confused with giant cell tumor of bone
Diagnosis: biopsy is required to obtain tissue diagnosis and institute therapy - improper biopsy techniques are associated with increased rates of complications
Biopsy should be performed by the surgeon responsible for definitive treatment of the sarcoma, or after discussion with this surgeon
Treatment
Operative: multi-agent chemotherapy and limb salvage resection
indications
high grade osteosarcoma
chemotherapy : preoperative chemotherapy given for 8-12 weeks followed by maintenance chemotherapy for 6-12 months after surgical resection - 98% necrosis after neo-adjuvant chemotherapy is good prognostic sign. Expression of multi-drug resistance (MDR) gene tends to have a poor prognosis as tumor cells can pump chemotherapy out of cell with MDR expression (present in 25% of primary lesions and 50% of metastatic lesions)
surgical technique - trend towards limb salvage whenever possible
overall survival in osteosarcoma is equal after limb salvage vs. amputation to deal with local extent of disease
rotationplasty is a great surgical option which optimizes the patient’s function, and most commonly done in a pediatric population
wide surgical resection - indicated in low grade osteosarcoma such as parosteal osteosarcoma
amputation
indications: pathologic fracture, encasing neurovascular bundle, enlarging during preop chemo AND adjacent to neurovascular bundle
A 68-year-old male sustains the humeral shaft fracture shown in Figures A and B. Upon presentation, he is unable to extend his thumb, fingers, and wrist. After 4 months of non-operative management, the fracture has healed, but his physical exam is unchanged. What is the next most appropriate step in management? R
EMG and nerve conduction tests followed by possible surgical exploration Continued observation Immediate surgical exploration Shoulder MRI CT scan of the humerus
The clinical presentation is consistent for a residual radial nerve palsy 4 months after a humeral shaft fracture. An EMG is indicated at this time to evaluate the status of the nerve recovery.
A radial nerve injury which occurs during a humeral shaft fracture or after bracing is not an indication for immediate exploration. Most often, the nerve function returns without surgical intervention. An EMG should be performed at 3-5 months to evaluate the status of the nerve recovery. If fasciculations are present, then this represents recovery, and observation should be continued. If fibrillations are present, this represents denervation, and surgical exploration should be considered.
Pollock et al followed 24 humeral-shaft fractures with associated radial-nerve injuries, 2 of which required open exploration and all recovered. They recommend careful observation for return of nerve function and exploration at 3.5-4 months after injury if there is still no clinical or EMG evidence of recovery.
Bostman et al reviewed 59 immediate and 16 secondary radial nerve palsies and no support emerged for routine early exploration in either group.
Figures A and B show an oblique fracture at the junction of the middle and distal 1/3 of the humeral shaft.
Illustration A shows the relative close position of the radial nerve to the humerus at the midlevel of humerus, and why it is at risk with a humerus shaft fracture.
Complications
Humeral shaft fx nonunion : no callous on radiograph and gross motion at the fracture site at 6 weeks from injury has a 100% PPV of going on to nonounion in closed humeral shaft fractures
Malunion: varus angulation is common but rarely has functional or cosmetic sequelae
risk factors: transverse fracture patterns
Radial nerve palsy
incidence: seen in 8-15% of closed fractures
increased incidence distal one-third fractures
neuropraxia most common injury in closed fractures and neurotomesis in open fractures
85-90% of improve with observation over 3 months
spontaneous recovery found at an average of 7 weeks, with full recovery at an average of 6 months
treatment
observation indicated as initial treatment in closed humerus fractures - obtain EMG at 3-4 months. Wrist extension in radial deviation is expected to be regained first - brachioradialis first to recover, extensor indicis is the last
surgical exploration
indications:
- open fracture with radial nerve palsy (likely neurotomesis injury to the radial nerve)
- closed fracture that fails to improve over ~ 3-6 months
fibrillations (denervation) seen at 3-4 months on EMG
A 33-year-old man requires a transfemoral amputation because of a mangling injury to his leg. Six months after the amputation he has persistent difficulty with ambulation because his distal femur moves into a subcutaneous position in his lateral thigh. It persists despite a well-fitted prosthesis. What technical error is the most likely cause of his dysfunction?
Inadequate posterior skin flap Inadequate anterior skin flap Failure to bevel the distal femur Lack of abductor myodesis to femur Lack of adductor myodesis to femur
Adductor myodesis is a critical part of a transfemoral amputation. If it is not performed, then the abductors and hip flexors can cause the femur to abduct, leading to severe problems with gait. The gait disturbance persists despite proper prosthetic fitting. A transfemoral amputation is usually performed with equal anterior and posterior flaps.
Pinzur et al highlight the fact that amputations are reconstructive procedures and should leave the patient with a functional residual limb.
Transfemoral Amputation:
- Maintain as much length as possible, however, ideal cut is 12 cm (10-15cm) above knee joint to allow for prosthetic fitting
Technique
- 5-10 degrees of adduction is ideal for improved prosthesis function
- adductor myodesis: improves clinical outcomes
creates dynamic muscle balance (otherwise have unopposed abductors). Provides soft tissue envelope that enhances prosthetic fitting
Gritti-Stokes amputation: amputation through the femur near level of adductor tubercle. Synovium is excised to prevent postoperative effusion. Patella is arthrodesed to the end of femur for improved end bearing
prepatellar soft tissue is maintained without iatrogenic injury - improved outcomes as compared to transfemoral amputation
Wound Healing Dependent on - vascular supply - nutritional status - immune status
Improved with
- albumin > 3.0 g/dL
- ischemic index > .5 measurement of doppler pressure at level being tested compared to brachial systolic pressure
- transcutaneous oxygen tension > 30 mm Hg (ideally 45 mm Hg)
- toe pressure > 40 mm Hg (will not heal if < 20 mm Hg)
ankle-brachial index (ABI) > 0.45
total lymphocyte count (TLC) > 1500/mm3
- Hyperbaric oxygen therapy
contraindications include: chemo or radiation therapy
pressure-sensitive implanted medical device (automatic implantable cardiac defibrillator, pacemaker, dorsal column stimulator, insulin pump) , undrained pneumothorax
Complications
- Wound healing
- Contractures: adjacent joint contractures are common
prevent with early aggressive mobilization and position changes
- Heterotopic ossification: more common in trauma-related setting
- Infection: trauma-related amputation have an infection rate of around 34%
- Postamputation Neuroma : occurs in 20-30% of amputees
prevent with proper nerve handling at the time of procedure. Treatment: targeted muscle rein nervation - a method of guiding neuronal regeneration to prevent or treat post-amputation neuroma pain and improve patient use of myoelectric prostheses
- Phantom limb pain
occurs in 53-100% of traumatic amputations
mirror therapy is a noninvasive treatment modality
- Bone overgrowth: most common complication with pediatric amputations. Treatment: prevent by performing disarticulation or using epihphyseal cap to cover medullary canal
In which of the following radiographs of different types of ankle fractures should the medial malleolus be treated with screw fixation directed parallel to the ankle joint?
Figure A: vertical medial mallelor fracture, supination adduction
Figure B: high fibular fracture
Figure C: supination external rotation
Figure D: pronation external rotation
Figure E: isolated medial malleolar fracture
Figure A show a classic SAD (supination adduction) fracture according to the Lauge-Hansen Classfication. This is evident by the vertical medial malleolar fracture and supinated position of the foot. The vertical medial malleolar fracture is best treated by screw fixation parallel to the joint (perpendicular to the fracture line). Careful attention must be paid to the presence of any medial plafond impaction from the talar displacement; if this is present, disimpaction and stabilization must be performed in order to optimize outcomes.
The referenced review article by Michelson covers rotational ankle fractures, with a review of the diagnosis, treatment options, and patient outcomes. He notes that unstable fractures (bimalleolar, bimalleolar equivalent, etc.) usually are managed with open reduction and internal fixation for optimal outcomes.
Incorrect answers:
Figure B shows a Weber C (high fibular) ankle fracture, PER, without any evidence of a medial malleolar fracture.
Figures C (SER IV), D (PER IV), and E (isloated medial malleolar fracture) all show fractures not suitable for screw fixation of the medial malleolus parallel to the joint since their fracture lines are not vertical.
Supination - Adduction (SA):
Talofibular sprain or distal fibular avulsion
Vertical medial malleolus and impaction of anteromedial distal tibia
Supination - External Rotation (SER):
Anterior tibiofibular ligament sprain
Lateral short oblique fibula fracture (anteroinferior to posterosuperior)
Posterior tibiofibular ligament rupture or avulsion of posterior malleolus
Medial malleolus transverse fracture or disruption of deltoid ligament
Pronation - Abduction (PA):
Medial malleolus transverse fracture or disruption of deltoid ligament
Anterior tibiofibular ligament sprain
Transverse comminuted fracture of the fibula above the level of the syndesmosis
Pronation - External Rotation (PER):
Medial malleolus transverse fracture or disruption of deltoid ligament
Anterior tibiofibular ligament disruption
Lateral short oblique or spiral fracture of fibula (anterosuperior to posteroinferior) above the level of the joint
Posterior tibiofibular ligament rupture or avulsion of posterior malleolus
In the treatment of intra-articular calcaneal fractures, surgical reduction and fixation has been shown to have improved outcomes over nonoperative treatment in all of the following patient groups EXCEPT:
Sedentary job Sanders IIb fractures Women Younger age (<29 years old) Previous calcaneus fracture
Previous calcaneus fracture
The referenced study by Buckley et al is a prospective study of intra-articular calcaneus fractures at several trauma centers. They found that overall, the outcomes after nonoperative treatment were not different from those after operative treatment. However, when stratifying groups, women who were managed operatively scored significantly higher on the SF-36 than did women who were managed nonoperatively. Also, patients who were less than twenty-nine years old, had a Böhler angle of 0 degrees to 14 degrees, a comminuted fracture, or a light workload did better after surgery compared with those who were treated non operatively.
incidence: most frequent tarsal fracture
17% open fractures
Pathophysiology
mechanism: traumatic axial loading is the primary mechanism of injury - fall from height, motor-vehicle accidents
pathoanatomy
- intra-articular fractures: primary fracture line results from oblique shear and leads to the following two primary fragments
1. superomedial fragment (constant fragment): includes the sustentaculum tali and is stabilized by strong ligamentous and capsular attachments
2. superolateral fragment: includes an intra-articular aspect through the posterior facet
secondary fracture lines dictate whether there is joint depression or tongue-type fracture
- extra-articular fractures: strong contraction of gastrocnemius-soleus with concomitant avulsion at its insertion site on calcaneus. More common in osteopenic bone
- anterior process fractures: inversion and plantar flexion of the foot cause avulsion of the bifurcate ligament
Associated injuries: extension into the calcaneocuboid joint occurs in 63%, vertebral injuries in 10%, contralateral calcaneus in 10%
Prognosis: poor with 40% complication rate
- increased due to mechanism (fall from height), smoking, and early surgery
- lateral soft tissue trauma increases the rate of complication
Osteology
articular facets
- superolateral fragment contains the articular facets
- superior articular surface contains three facets that articulate with the talus
- posterior facet is the largest and is the major weight bearing surface - the flexor hallucis longus tendon runs just inferior to it and can be injured with errant drills/screws that are too long
- middle facet is anteromedial on sustentaculum tali
- anterior facet is often confluent with middle facet
sinus tarsi: between the middle and posterior facets lies the interosseous sulcus (calcaneal groove) that together with the talar sulcus makes up the sinus tarsi
sustentaculum tali: projects medially and supports the neck of talus. FHL passes beneath it. Deltoid and talocalcaneal ligament connect it to the talus. Contained in the anteromedial fragment, which remains “constant” due to medial talocalcaneal and interosseous ligaments
bifurcate ligament: connects the dorsal aspect of the anterior process to the cuboid and navicular
Classification
Extra-articular (25%) - avulsion injury of anterior process by bifurcate ligament, sustentaculum tali, calcaneal tuberosity (Achilles tendon avulsion)
Intra-articular (75%): Essex-Lopresti classification - the primary fracture line runs obliquely through the posterior facet forming two fragments. The secondary fracture line runs in one of two planes
- the axial plane beneath the facet exiting posteriorly in tongue-type fractures when the superolateral fragment and posterior facet remain attached to the tuberosity posteriorly
- behind the posterior facet in joint depression fractures
Sanders classification
Based on the number of articular fragments seen on the coronal CT image at the widest point of the posterior facet
Type I • Nondisplaced posterior facet (regardless of number of fracture lines)
Type II • One fracture line in the posterior facet (two fragments)
Type III • Two fracture lines in the posterior facet (three fragments)
Type IV • Comminuted with more than three fracture lines in the posterior facet (four or more fragments)
Physical exam: diffuse tenderness to palpation, ecchymosis and swelling, shortened, widened, heel with a varus deformity
Imaging
Radiographs: AP, lateral, oblique
Optional views:
Broden: allows visualization of posterior facet - useful for evaluation of intraoperative reduction of posterior facet
with ankle in neutral dorsiflexion and ~45 degrees internal rotation, take x-rays at 40, 30, 20, and 10 degrees cephalad from neutral
Harris : visualizes tuberosity fragment widening, shortening, and varus positioning. Place the foot in maximal dorsiflexion and angle the x-ray beam 45 degrees
AP ankle: demonstrates lateral wall extrusion causing fibular impingement
Findings
- double-density sign: represents subtalar incongruity - indicates partial separation of facet from sustentaculum
- calcaneal shortening
- varus tuberosity deformity
- decreased Bohler angle : angle between line from highest point of anterior process to highest point of posterior facet + line tangential to superior edge of tuberosity
measured on lateral view (normal 20-40° ) represents collapse of the posterior facet
- increased angle of Gissane : angle between line along lateral margin of posterior facet + line anterior to beak of calcaneus - measured on lateral view (normal 120-145°) represents collapse of the posterior facet
CT: gold standard - demonstrates posterior and middle facet displacement, calcaneocuboid joint involvement and tuberosity displacement
MRI: used only to diagnose calcaneal stress fractures in the presence of normal radiographs and/or uncertain diagnosis
Treatment
Nonoperative:
- Cast immobilization with nonweightbearing for 6 weeks. Indications: calcaneal stress fractures
- Cast immobilization with nonweightbearing for 10 to 12 weeks. Indications: small extra-articular fracture (<1 cm) with intact Achilles tendon and <2 mm displacement, Sanders Type I (non displaced), anterior process fracture involving <25% of calcaneocuboid joint, comorbidities that preclude good surgical outcome (smoker, diabetes, PVD)
techniques: begin early range of motion exercises once swelling allows
Operative
Closed reduction with percutaneous pinning
indications
- minimally displaced tongue-type fxs or those with mild shortening
- large extra-articular fractures (>1 cm)
Early reduction prevents skin sloughing and need for subsequent flap coverage
techniques: lag screws from posterior superior tuberosity directed inferior and distal
ORIF
indications
- displaced tongue-type fractures
- large extra-articular fractures (>1 cm) with detachment of Achilles tendon and/or > 2 mm displacement (urgent if skin is compromised)
- Sanders Type II and III (posterior facet displacement >2 to 3 mm, flattening of Bohler angle, or varus malalignment of the tuberosity)
- anterior process fracture with >25% involvement of calcaneocuboid joint
- displaced sustentaculum fractures
timing: wait 10-14 days until swelling and blisters resolve and wrinkle sign present 10-14 days. No benefit to early surgery due to significant soft tissue swelling
outcomes
surgical outcome correlates with the number of intra-articular fragments and the quality of articular reduction
factors associated with a poor outcome: age > 50, obesity, manual labor, workers comp, smokers, bilateral calcaneal fractures, multiple trauma, vasculopathies, men do worse with surgery than women
Factors associated with most likely need for a secondary subtalar fusion: male worker’s compensation patient who participates in heavy labor work with an initial Böhler angle less than 0 degrees
Primary subtalar arthrodesis
indications: Sanders Type IV
techniques: combined with ORIF to restore height
A 45-year-old male is involved in a motor vehicle accident and sustains the injury described below. Which of the following is the most appropriate approach for surgical fixation of this fracture?
Fig A: disruption ilioischial line, intact iliopectineal. Axial CT: horizontal orientation of column fracture.
Ilioinguinal Kocher-Langenbeck Stoppa Stoppa with lateral window Extended iliofemoral
The images demonstrate a posterior column acetabular fracture. These are best surgically treated with a Kocher-Langenbeck approach, which allows access to the posterior column and posterior wall. Figure A shows disruption of the ilioischial line with an intact iliopectineal line which is diagnostic of this fracture pattern. The CT image in Figure D shows the characteristic horizontal (coronal) orientation of the column fracture when viewed on an axial CT. Illustration A shows the radiographic landmarks used in diagnosing acetabular fractures. Illustrations B and C show the orientation of column and wall fractures respectively. Ilioinguinal and Stoppa approaches allow access for anterior column fixation and symphysis fixation respectively. The extended iliofemoral approach can be used to treat both column injuries, but has high rates of post-operative heterotopic ossification.
Acetabulum fractures can involve one or more of the two columns, two walls or roof within the pelvis
Osteology
acetabular inclination & ante version: mean lateral inclination of 40 to 48 degrees, anteversion of 18 to 21 degrees
Column theory : acetabulum is supported by two columns of bone - form an “inverted Y”. Connected to sacrum through sciatic buttress
- Posterior column: comprised of quadrilateral surface, posterior wall and dome, ischial tuberosity, greater/lesser sciatic notches
- anterior column: comprised of anterior ilium (gluteus medius tubercle), anterior wall and dome, iliopectineal eminence, lateral superior pubic ramus
Vascular
corona mortis : anastomosis of external iliac (epigastric) and internal iliac (obturator) vessels - at risk with lateral dissection over superior pubic ramus
Letournel Classification
Judet and Letournel: most common referenced classification system - classifed as 5 elementary and 5 associated fracture patterns
Most common fracture patterns
Younger : posterior wall or transverse fracture “family”
(transverse, T-type, transverse + posterior wall)
Elderly : anterior column (e.g., quadrilateral plate fractures), anterior column, posterior hemitransverse, assoicated both column fractures
Posterior wall
• Most common
• “gull sign” on obturator oblique view
Posterior column
• check for injury to superior gluteal NV bundle
Anterior wall
• Very rare
Anterior column
• More common in elderly patients with fall from standing (most common in elderly is “anterior column + medial wall”)
Transverse
• Axial CT shows anterior to posterior fx line
• Only elementary fx to involve both columns
Associated Both Column
• Characterized by dissociation of the articular surface from the inonimate bone
• “spur sign” on obturator oblique
Transverse + Post. Wall
• Most common associated fx
T Shaped
• May need combined approach
Anterior column or wall + Post. hemitransverse
• Common in elderly patients
Post. column + Post. wall
• Only associated fracture that does not involve both columns
Treatment
Nonoperative: protected weight bearing for 6-8 weeks
indications: patient factors - high operative risk (e.g., elderly patients, presence of DVT), morbid obesity, open contaminated wound, late presenting > 3 weeks
fracture characteristics: minimally displaced fracture (< 2 mm), < 20% posterior wall fractures (treatment based on size of posterior wall is controversial - recommend an exam under anesthesia (EUA) using fluoroscopy best method to test stability), Femoral head congruency with weight bearing roof (out of traction) - both column fracture pattern with secondary congruence (out of traction), displaced fracture with roof arcs > 45° in AP and Judet views or >10 mm on axial CT cuts
technique
- Skeletal traction NOT required if stable fracture pattern, outside the weight-bearing dome
- activity as tolerated with crutches/walker
Weight-bearing: lowest joint reactive forces seen with toe-touch weight bearing and passive hip abduction, greatest joint contact force seen when rising from a chair on the affected extremity
DVT prophylaxis if slow to mobilize
Close radiographic follow-up
Operative treatment
open reduction and internal fixation
indications
- Patient factors: < 3 weeks from date of injury, physiologically stable, adequate soft-tissue envelope, no local infection, pregnancy is not contraindication to surgical fixation
- Fracture factors: displacement of roof (> 2 mm), unstable fracture pattern (e.g. posterior wall fracture involving > 40-50%), marginal impaction, intra-articular loose bodies, irreducible fracture-dislocation
Approaches
- Anterior : ilioinguinal, iliofemoral, modified stoppa
- Posterior: Kocher-Langenbach
- Combined: extended ilifemoral
Fixation modalities:
- Column fixation strategies: reconstruction bridging plate and screws, percutaneous column screws, cable fixation
- Wall fixation strategies: bridge plate and screws, lag screw and neutralization plate, spring (butress) plate
outcomes
timing: associated hip dislocations should be reduced within 12 hours for improved outcomes
worse outcomes with fixation of fracture > 3 weeks from time of injury - earlier operative treatment associated with increased chance of anatomic reduction
Peri-operative : clinical outcome correlates with quality of articular reduction
Postoperative CT scan is most accurate way to determine posterior wall accuracy of reduction which has greatest correlation with clinical outcome - ideally articular reduction <2mm
Post-operative
greatest stress on acetabular repair occurs when rising from a seated position using the affected leg, and occurs in the posterior superior portion of the acetabulum
Functional outcomes most strongly correlate with hip muscle strength and restoration of gait postoperatively
Total hip arthroplasty
indications
usually elderly patients with significant osteopenia and/or significant comminution, pre-existing arthritis, post-traumatic arthritis in all ages
techniques
timing: immediate vs. delayed THA
immediate THA (with, or without, fracture fixation)
wall fractures : butress plate with multi-hole cup
column fracture: cage and cup constructs
delayed THA
outcomes: patients older than 60 years have approx. a 30% late conversion rate to THA after acetabular fractures
10-year implant survival noted to be around 75-80%
During the ilioinguinal approach to the pelvis, the corona mortis artery must be identified and ligated if present. The corona mortis artery joins the external illiac artery with which other major artery?
Pudendal Deep illiac circumflex Hypogastric Obturator Testicular
The “corona mortis” (translated as “crown of death”) artery is a vascular variant that joins the external illiac and the obturator artery as it crosses the superior pubic ramus. Tornetta et al did a study where “fifty cadaver halves were dissected to determine the occurrence and location of the corona mortis. Anastomoses between the obturator and external iliac systems occurred in 84% of the specimens. Thirty-four percent had an arterial connection, 70% had a venous connection, and 20% had both. The distance from the symphysis to the anastomotic vessels averaged 6.2 cm (range, 3-9 cm).” The corona mortis can be injured in superior ramus fractures and iatrogenically while plating pelvic ring injuries using the ilioinguinal approach.
Acetabulum fractures can involve one or more of the two columns, two walls or roof within the pelvis
Osteology
acetabular inclination & ante version: mean lateral inclination of 40 to 48 degrees, anteversion of 18 to 21 degrees
Column theory : acetabulum is supported by two columns of bone - form an “inverted Y”. Connected to sacrum through sciatic buttress
- Posterior column: comprised of quadrilateral surface, posterior wall and dome, ischial tuberosity, greater/lesser sciatic notches
- anterior column: comprised of anterior ilium (gluteus medius tubercle), anterior wall and dome, iliopectineal eminence, lateral superior pubic ramus
Vascular
corona mortis : anastomosis of external iliac (epigastric) and internal iliac (obturator) vessels - at risk with lateral dissection over superior pubic ramus
Letournel Classification
Judet and Letournel: most common referenced classification system - classifed as 5 elementary and 5 associated fracture patterns
Most common fracture patterns
Younger : posterior wall or transverse fracture “family”
(transverse, T-type, transverse + posterior wall)
Elderly : anterior column (e.g., quadrilateral plate fractures), anterior column, posterior hemitransverse, assoicated both column fractures
Posterior wall
• Most common
• “gull sign” on obturator oblique view
Posterior column
• check for injury to superior gluteal NV bundle
Anterior wall
• Very rare
Anterior column
• More common in elderly patients with fall from standing (most common in elderly is “anterior column + medial wall”)
Transverse
• Axial CT shows anterior to posterior fx line
• Only elementary fx to involve both columns
Associated Both Column
• Characterized by dissociation of the articular surface from the inonimate bone
• “spur sign” on obturator oblique
Transverse + Post. Wall
• Most common associated fx
T Shaped
• May need combined approach
Anterior column or wall + Post. hemitransverse
• Common in elderly patients
Post. column + Post. wall
• Only associated fracture that does not involve both columns
Treatment
Nonoperative: protected weight bearing for 6-8 weeks
indications: patient factors - high operative risk (e.g., elderly patients, presence of DVT), morbid obesity, open contaminated wound, late presenting > 3 weeks
fracture characteristics: minimally displaced fracture (< 2 mm), < 20% posterior wall fractures (treatment based on size of posterior wall is controversial - recommend an exam under anesthesia (EUA) using fluoroscopy best method to test stability), Femoral head congruency with weight bearing roof (out of traction) - both column fracture pattern with secondary congruence (out of traction), displaced fracture with roof arcs > 45° in AP and Judet views or >10 mm on axial CT cuts
technique
- Skeletal traction NOT required if stable fracture pattern, outside the weight-bearing dome
- activity as tolerated with crutches/walker
Weight-bearing: lowest joint reactive forces seen with toe-touch weight bearing and passive hip abduction, greatest joint contact force seen when rising from a chair on the affected extremity
DVT prophylaxis if slow to mobilize
Close radiographic follow-up
Operative treatment
open reduction and internal fixation
indications
- Patient factors: < 3 weeks from date of injury, physiologically stable, adequate soft-tissue envelope, no local infection, pregnancy is not contraindication to surgical fixation
- Fracture factors: displacement of roof (> 2 mm), unstable fracture pattern (e.g. posterior wall fracture involving > 40-50%), marginal impaction, intra-articular loose bodies, irreducible fracture-dislocation
Approaches
- Anterior : ilioinguinal, iliofemoral, modified stoppa
- Posterior: Kocher-Langenbach
- Combined: extended ilifemoral
Fixation modalities:
- Column fixation strategies: reconstruction bridging plate and screws, percutaneous column screws, cable fixation
- Wall fixation strategies: bridge plate and screws, lag screw and neutralization plate, spring (butress) plate
outcomes
timing: associated hip dislocations should be reduced within 12 hours for improved outcomes
worse outcomes with fixation of fracture > 3 weeks from time of injury - earlier operative treatment associated with increased chance of anatomic reduction
Peri-operative : clinical outcome correlates with quality of articular reduction
Postoperative CT scan is most accurate way to determine posterior wall accuracy of reduction which has greatest correlation with clinical outcome - ideally articular reduction <2mm
Post-operative
greatest stress on acetabular repair occurs when rising from a seated position using the affected leg, and occurs in the posterior superior portion of the acetabulum
Functional outcomes most strongly correlate with hip muscle strength and restoration of gait postoperatively
Total hip arthroplasty
indications
usually elderly patients with significant osteopenia and/or significant comminution, pre-existing arthritis, post-traumatic arthritis in all ages
techniques
timing: immediate vs. delayed THA
immediate THA (with, or without, fracture fixation)
wall fractures : butress plate with multi-hole cup
column fracture: cage and cup constructs
delayed THA
outcomes: patients older than 60 years have approx. a 30% late conversion rate to THA after acetabular fractures
10-year implant survival noted to be around 75-80%
An 18-year-old football player is injured after making a tackle with his left shoulder. He has decreased sensation over the lateral aspect of the left shoulder and radial aspect of the forearm. Motor examination reveals weakness to shoulder abduction and external rotation as well as elbow flexion. He has decreased reflexes of the biceps tendon on the left side but full, nontender range of motion of the cervical spine. What anatomic site has been injured?
Fourth cervical nerve root Upper trunk of the brachial plexus Middle trunk of the brachial plexus Lateral cord of the brachial plexus Axillary nerve
The athlete has symptoms referable to the axillary, musculocutaneous, and suprascapular nerves resulting from an injury to the upper trunk of the brachial plexus. This portion of the plexus is formed by contributions of the fourth through sixth cervical nerve roots. This area is often contused or stretched following a tackling maneuver that results in either depression of the shoulder from contact at Erb’s point or traction of the upper plexus from forced stretching of the neck to the contralateral side.
Supraclavicular injuries:
- complete involvement of all roots is most common: 75%-80% of traumatic BPIs
- C5 and C6 upper trunk (Erb palsy): 20%-25% of traumatic BPIs
- C8, T1 or lower (Klumpke palsy): 0.6%-3.0% of traumatic BPIs
Mechanism
- high speed vehicular accidents (mostly motorcycle): 83% of traumatic BPIs
- caudally forced shoulder: predominantly affect upper brachial plexus - with high enough energy all roots can be affected
- forced arm abduction (as in grabbing onto something while falling) predominantly affects lower roots
Prognosis
Recovery of reconstructed plexus can take up to 3 years
nerve regeneration occurs at speed of 1mm/day
infraclavicular plexus injuries have better prognosis than supraclavicular injuries Upper plexus injuries have improved prognosis - preservation of hand function root avulsion (preganglionic injuries) have worst prognosis (not repairable) other surgeries such as arthrodesis and tendon transfers may be needed
Anatomy
brachial plexus motor and sensory innervation
Classification
Preganglionic vs. postganglionic
- preganglionic: avulsion proximal to dorsal root ganglion
involves CNS which does not regenerate – little potential recovery of motor function (poor prognosis)
Lesions suggesting preganglionic injury:
Horner’s syndrome (disruption of sympathetic chain), winged scapula medially, loss of serratus anterior (long thoracic nerve) rhomboids (dorsal scapular nerve) leads to medial winging (inferior border goes medial). Presents with motor deficits (flail arm), sensory intact, absence of a Tinel sign or tenderness to percussion in the neck, normal histamine test (C8-T1 sympathetic ganglion), intact triple response (redness, wheal, flare), elevated hemidiaphragm (phrenic nerve
rhomboid paralysis (dorsal scapular nerve), supraspinatus/infraspinatus (suprascapular nerve), latissimus dorsi (thoracodorsal)
Evaluation: EMG may show loss of innervation to cervical paraspinals
Postganglionic: involve PNS, capable of regeneration (better prognosis)
Presents with motor deficit (flail arm), sensory deficits
Evaluation: EMG shows maintained innervation to cervical paraspinals
abnormal histamine test
only redness and wheal, but NO flare
Classification based on location
Upper Lesion: Erb’s Palsy (C5,6)
Most common obstetric brachial plexopathy
Results from excessive displacement of head to opposite side and depression of shoulder on same side producing traction on plexus. Occurs during difficult delivery in infants or fall onto shoulder in adults
Best prognosis
Clinically, arm will be adducted, internally rotated, at shoulder; pronated, extended at elbow (“waiter’s tip”)
C5 deficiency: axillary nerve deficiency (weakness in deltoid, teres minor), suprascapular nerve deficiency (weakness in supraspinatus, infraspinatus), musculocutaneous nerve deficiency (weakness to biceps)
C6 deficiency: radial nerve deficiency (weakness in brachioradialis, supinator)
Lower Lesion: Klumpke Palsy (C8,T1)
Rare in obstetric palsy. Usually avulsion injuries caused by excessive abduction (person falling from height clutching on object to save himself)
Other causes may include cervical rib, or lung mets in lower deep cervical lymph nodes
Frequently associated with a preganglion injury and Horner’s Syndrome
Poor prognosis
Deficit of all of the small muscles of the hand (ulnar and median nerves) - Clinically, presents as “claw hand” with wrist held in extreme extension because of the unopposed wrist extensors. Hyperextension of MCP due to loss of hand intrinsics, flexion of IP joints due to loss of hand intrinsics
Total Palsy (C5-T1): A form of brachial plexopathy
Worst prognosis - Leads to a flaccid arm
Involves both motor and sensory
Which one of the following lower extremity amputations requires a soft-tissue balancing procedure to prevent deformity following amputation?
Gritti-Stokes Transtibial Transmetatarsal Knee disarticulation Lisfranc
The Lisfranc level amputation removes the attachment of the peroneus brevis (base of the fifth metatarsal) and the peroneus longus (base of the first metatarsal), creating a varus deformity due to unopposed overpull by the tibialis anterior and posterior muscles. An anterior tibialis tendon transfer may be necessary. Other possible tissue balancing choices include posterior tibialis transfer and lengthening of the gastrocsoleus complex. Another option is to leave the base of the fifth metatarsal attached to preserve the eversion pull of the peroneus brevis. All the other amputations do not require soft-tissue balancing procedures to prevent deformities. However, adherence to the prescribed surgical techniques for reattachment of major muscle groups is important for optimizing limb strength and function.
Ankle/Foot Amputation:
Syme amputation (ankle disarticulation) - patent tibialis posterior artery is required. More energy efficient than midfoot even though it is more proximal
Stable heel pad is most important factor
Used successfully to treat forefoot gangrene in diabetics
Technique: medial and lateral malleoli are removed flush with distal tibia articular surface. The medial and lateral flares of the tibia and fibula are beveled to enhance heel pad adherence. Heel pad is secured to anterior tibia
Pirogoff amputation (hindfoot amputation): removal of the forefoot and talus followed by calcaneotibial arthrodesis
calcaneus is osteotomized and rotated 50-90 degrees to keep posterior aspect of calcaneus distal
Allows patient to mobilize independently without use of prosthetic
Chopart or Boyd amputation (hindfoot amputation)
a partial foot amputation through the talonavicular and calcaneocuboid joints. Primary complication is equinus deformity - avoid by lengthening of the Achilles tendon and transfer of the tibialis anterior to the talar neck
leads to apropulsive gait pattern because the amputation is unable to support modern dynamic elastic response prosthetic feet
Lisfranc amputation (midfoot amputation) - equinovarus deformity is common caused by unopposed pull of tibialis posterior and gastroc/soleus. Prevent by maintaining insertion of peroneus brevis and performing achilles lengthening. A walking cast is generally used for 4 week to prevent late equinus contracture. Energy cost of walking similar to that of BKA
Transmetatarsal amputation: more appealing to patients who refuse transtibial amputations
Almost all require achilles lengthening to prevent equinus
Great toe amputations: preserve 1cm at base of proximal phalanx - preserves insertion of plantar fascia, sesamoids, and flexor hallucis brevis - reduces amount of weight transfer to remaining toes so lessens risk of ulceration
Metabolic cost of walking
- increases with more proximal amputations
perform amputations at lowest possible level to preserve function
- exception: Syme amputation is more efficient than midfoot amputation
- inversely proportional to length of remaining limb
Ranking of metabolic demand (% represents amount of increase compared to baseline)
Syme - 15%
transtibial: traumatic - 25% average, short BKA - 40%, long BKA - 10%, vascular - 40%
transfemoral: traumatic - 68%, vascular - 100%
thru-knee amputation: varies based on patient habitus but is somewhere between transtibial and transfemoral
Most proximal amputation level available in children to maintain walking speeds without increased energy expenditure compared to normal children
Transfemoral Amputation:
- Maintain as much length as possible, however, ideal cut is 12 cm (10-15cm) above knee joint to allow for prosthetic fitting
Technique
- 5-10 degrees of adduction is ideal for improved prosthesis function
- adductor myodesis: improves clinical outcomes
creates dynamic muscle balance (otherwise have unopposed abductors). Provides soft tissue envelope that enhances prosthetic fitting
Gritti-Stokes amputation: amputation through the femur near level of adductor tubercle. Synovium is excised to prevent postoperative effusion. Patella is arthrodesed to the end of femur for improved end bearing
prepatellar soft tissue is maintained without iatrogenic injury - improved outcomes as compared to transfemoral amputation
Wound Healing Dependent on - vascular supply - nutritional status - immune status
Improved with
- albumin > 3.0 g/dL
- ischemic index > .5 measurement of doppler pressure at level being tested compared to brachial systolic pressure
- transcutaneous oxygen tension > 30 mm Hg (ideally 45 mm Hg)
- toe pressure > 40 mm Hg (will not heal if < 20 mm Hg)
ankle-brachial index (ABI) > 0.45
total lymphocyte count (TLC) > 1500/mm3
- Hyperbaric oxygen therapy
contraindications include: chemo or radiation therapy
pressure-sensitive implanted medical device (automatic implantable cardiac defibrillator, pacemaker, dorsal column stimulator, insulin pump) , undrained pneumothorax
Complications
- Wound healing
- Contractures: adjacent joint contractures are common
prevent with early aggressive mobilization and position changes
- Heterotopic ossification: more common in trauma-related setting
- Infection: trauma-related amputation have an infection rate of around 34%
- Postamputation Neuroma : occurs in 20-30% of amputees
prevent with proper nerve handling at the time of procedure. Treatment: targeted muscle rein nervation - a method of guiding neuronal regeneration to prevent or treat post-amputation neuroma pain and improve patient use of myoelectric prostheses
- Phantom limb pain
occurs in 53-100% of traumatic amputations
mirror therapy is a noninvasive treatment modality
- Bone overgrowth: most common complication with pediatric amputations. Treatment: prevent by performing disarticulation or using epihphyseal cap to cover medullary canal
Which of the following is most important to achieve a good outcome following a Syme amputation?
trimming any dog ears a viable and stable heel pad achilles tendon lengthening preserving the malleoli tenodesing the extensor digitorum longus to the tibial shaft
A Syme amputation is effectively a tibiotalar disarticulation, which provides an end-bearing stump that could potentially allow ambulation without a prosthesis over short distances. It works better for tumor and trauma, but the heel pad must be viable. The two most common problems are 1) skin sloughing from compromised vascular supply and 2) migration of the heel pad due to instability. A hypermobile heel pad can cause difficulty with prosthesis wear and damage to the soft tissues which can eventually lead to failure. Both malleoli are usually removed in the procedure, except in children or during the first stage procedure of a diabetic or infection case. The tibialis anterior is usually tenodesed to the anterior heel pad along with the EDL tendon to avoid posterior migration of the heel pad.
Ankle/Foot Amputation:
Syme amputation (ankle disarticulation) - patent tibialis posterior artery is required. More energy efficient than midfoot even though it is more proximal
Stable heel pad is most important factor
Used successfully to treat forefoot gangrene in diabetics
Technique: medial and lateral malleoli are removed flush with distal tibia articular surface. The medial and lateral flares of the tibia and fibula are beveled to enhance heel pad adherence. Heel pad is secured to anterior tibia
Pirogoff amputation (hindfoot amputation): removal of the forefoot and talus followed by calcaneotibial arthrodesis
calcaneus is osteotomized and rotated 50-90 degrees to keep posterior aspect of calcaneus distal
Allows patient to mobilize independently without use of prosthetic
Chopart or Boyd amputation (hindfoot amputation)
a partial foot amputation through the talonavicular and calcaneocuboid joints. Primary complication is equinus deformity - avoid by lengthening of the Achilles tendon and transfer of the tibialis anterior to the talar neck
leads to apropulsive gait pattern because the amputation is unable to support modern dynamic elastic response prosthetic feet
Lisfranc amputation (midfoot amputation) - equinovarus deformity is common caused by unopposed pull of tibialis posterior and gastroc/soleus. Prevent by maintaining insertion of peroneus brevis and performing achilles lengthening. A walking cast is generally used for 4 week to prevent late equinus contracture. Energy cost of walking similar to that of BKA
Transmetatarsal amputation: more appealing to patients who refuse transtibial amputations
Almost all require achilles lengthening to prevent equinus
Great toe amputations: preserve 1cm at base of proximal phalanx - preserves insertion of plantar fascia, sesamoids, and flexor hallucis brevis - reduces amount of weight transfer to remaining toes so lessens risk of ulceration
Metabolic cost of walking
- increases with more proximal amputations
perform amputations at lowest possible level to preserve function
- exception: Syme amputation is more efficient than midfoot amputation
- inversely proportional to length of remaining limb
Ranking of metabolic demand (% represents amount of increase compared to baseline)
Syme - 15%
transtibial: traumatic - 25% average, short BKA - 40%, long BKA - 10%, vascular - 40%
transfemoral: traumatic - 68%, vascular - 100%
thru-knee amputation: varies based on patient habitus but is somewhere between transtibial and transfemoral
Most proximal amputation level available in children to maintain walking speeds without increased energy expenditure compared to normal children
Transfemoral Amputation:
- Maintain as much length as possible, however, ideal cut is 12 cm (10-15cm) above knee joint to allow for prosthetic fitting
Technique
- 5-10 degrees of adduction is ideal for improved prosthesis function
- adductor myodesis: improves clinical outcomes
creates dynamic muscle balance (otherwise have unopposed abductors). Provides soft tissue envelope that enhances prosthetic fitting
Gritti-Stokes amputation: amputation through the femur near level of adductor tubercle. Synovium is excised to prevent postoperative effusion. Patella is arthrodesed to the end of femur for improved end bearing
prepatellar soft tissue is maintained without iatrogenic injury - improved outcomes as compared to transfemoral amputation
Wound Healing Dependent on - vascular supply - nutritional status - immune status
Improved with
- albumin > 3.0 g/dL
- ischemic index > .5 measurement of doppler pressure at level being tested compared to brachial systolic pressure
- transcutaneous oxygen tension > 30 mm Hg (ideally 45 mm Hg)
- toe pressure > 40 mm Hg (will not heal if < 20 mm Hg)
ankle-brachial index (ABI) > 0.45
total lymphocyte count (TLC) > 1500/mm3
- Hyperbaric oxygen therapy
contraindications include: chemo or radiation therapy
pressure-sensitive implanted medical device (automatic implantable cardiac defibrillator, pacemaker, dorsal column stimulator, insulin pump) , undrained pneumothorax
Complications
- Wound healing
- Contractures: adjacent joint contractures are common
prevent with early aggressive mobilization and position changes
- Heterotopic ossification: more common in trauma-related setting
- Infection: trauma-related amputation have an infection rate of around 34%
- Postamputation Neuroma : occurs in 20-30% of amputees
prevent with proper nerve handling at the time of procedure. Treatment: targeted muscle rein nervation - a method of guiding neuronal regeneration to prevent or treat post-amputation neuroma pain and improve patient use of myoelectric prostheses
- Phantom limb pain
occurs in 53-100% of traumatic amputations
mirror therapy is a noninvasive treatment modality
- Bone overgrowth: most common complication with pediatric amputations. Treatment: prevent by performing disarticulation or using epihphyseal cap to cover medullary canal
A 32-year-old laborer reports left ankle pain and deformity. History reveals that he sustained a left ankle fracture 2 years ago and was treated with closed reduction and casting. Radiographs are shown in Figures 25a through 25c. What is the most appropriate management? R
FIGURES: A B C : fibular and medial malleloar non-union, talar shift >1mm
Bracing and physical therapy
Intra-articular injection of steroids into the ankle joint, bracing, and physical therapy
Intra-articular injection of hyaluronic acid product into the ankle joint, bracing, and physical therapy
Ankle fusion
Corrective osteotomy of the fibula and medial malleolus with reconstruction of the syndesmosis if unstable
Corrective osteotomy of fibular malunions, with appropriate lengthening, even in the presence of early arthritis, has been shown to decrease ankle pain and increase stability. Reduction and bone grafting of the medial malleolar nonunion is also needed. There is no evidence supporting the use of intra-articular steroids or hyaluronic acid in the ankle joint. Lateral talar displacement of even 1 mm has been reproducibly shown to decrease tibiotalar contact by 40% to 42%, causing a predisposition to arthritis.
Supination - Adduction (SA):
Talofibular sprain or distal fibular avulsion
Vertical medial malleolus and impaction of anteromedial distal tibia
Supination - External Rotation (SER):
Anterior tibiofibular ligament sprain
Lateral short oblique fibula fracture (anteroinferior to posterosuperior)
Posterior tibiofibular ligament rupture or avulsion of posterior malleolus
Medial malleolus transverse fracture or disruption of deltoid ligament
Pronation - Abduction (PA):
Medial malleolus transverse fracture or disruption of deltoid ligament
Anterior tibiofibular ligament sprain
Transverse comminuted fracture of the fibula above the level of the syndesmosis
Pronation - External Rotation (PER):
Medial malleolus transverse fracture or disruption of deltoid ligament
Anterior tibiofibular ligament disruption
Lateral short oblique or spiral fracture of fibula (anterosuperior to posteroinferior) above the level of the joint
Posterior tibiofibular ligament rupture or avulsion of posterior malleolus
A 37-year-old man presents to the emergency room with the left lower extremity injury shown in Figure A. A radiograph is shown in Figure B. Which of the following has the most impact on the decision to attempt limb salvage versus amputation?
FIGURES: A & B - open distal third tibia spiral fracture, with high fib fracture
Quality of initial fracture reduction History of tobacco use Insurance status Extent of soft tissue injury Operative debridement and irrigation within 1 hour of injury
Extent of soft tissue injury has been shown in Level 2 evidence as having the highest impact on the decision to undergo limb salvage or amputation.
The referenced study by MacKenzie et al looked at 527 of the 601 patients initially enrolled in the Lower Extremity Assessment Project (LEAP) and looked at several variables which are thought to be predictors of amputation. Severe muscle injury had the highest impact on the decision to amputate the limb, likely related to the surgeon’s assessment that the salvaged limb would function poorly because of the risk of infection, nonunion, and poor function. The absence of plantar sensation had the next most significant impact on surgical decision making. Factors that would influence proceeding with an amputation include an nonviable limb, irreparable vascular injury, warm ischemia time of more than 8 hours, or a severe crush injury with minimal remaining viable tissue. Amputation should also be considered when attempts at limb salvage leave the limb so severely damaged that function will be less satisfactory than that afforded by a prosthetic replacement, are a threat to the patient’s life, or would demand multiple surgical procedures and prolonged reconstruction time that is incompatible with the personal, sociologic, and economic consequences the patient is willing to undergo.
amputation vs. reconstruction
LEAP study
◾impact on decision to amputate limb: severe soft tissue injury - highest impact on decision-making process
◾absence of plantar sensation - 2nd highest impact on surgeon’s decision making process. Not an absolute contraindication to reconstruction. Plantar sensation can recover by long-term follow-up
Outcome measure
◾SIP (sickness impact profile) and return to work not significantly different between amputation and reconstruction at 2 years in limb-threatening injuries
◾25% infection rate
◾Mangled foot and ankle injuries requiring free tissue transfer have a worse SIP than BKA
◾most important factor to determine patient-reported outcome is the ability to return to work (About 50% of patients are able to return to work )
METALS study
◾study focused on military population in response to LEAP study
◾slightly better results in regard to patient-reported outcomes for the amputation group with a lower risk of PTSD - more severe limbs were going into salvage pathway, military population with better access to prostheses
A 25-year-old male is involved in a motor vehicle accident and presents with the injury shown in Figure A. Early fixation of this fracture pattern is associated with all of the following EXCEPT?
FIGURES: A: acetabular fracture
Decreased length of hospital stay Improved functional outcome Greater organ dysfunction Higher likelihood of being discharged to home as opposed to a rehab facility Improved fracture reduction
Early fixation of acetabular fractures is associated with lesser organ dysfunction, so therefore answer three is not true.
Plaisier et al showed the timing of acetabular and pelvic ring fracture fixation greatly impacted patient outcome. Patients who had fixation within 24 hours of injury showed shorter length of stay in the hospital and ICU (decreased number of ventilator days), improved functional outcomes including a highly likelihood of being discharged to home as opposed to a rehabilitation facility, and lesser organ dysfunction.
The reference by Matta et al is a classic article that shows that patients fixed within 3 weeks of injury showed both a higher rate of anatomical reduction and lower overall complication rate than patients with similar fracture patterns treated after 3 weeks.
Acetabulum fractures can involve one or more of the two columns, two walls or roof within the pelvis
Osteology
acetabular inclination & ante version: mean lateral inclination of 40 to 48 degrees, anteversion of 18 to 21 degrees
Column theory : acetabulum is supported by two columns of bone - form an “inverted Y”. Connected to sacrum through sciatic buttress
- Posterior column: comprised of quadrilateral surface, posterior wall and dome, ischial tuberosity, greater/lesser sciatic notches
- anterior column: comprised of anterior ilium (gluteus medius tubercle), anterior wall and dome, iliopectineal eminence, lateral superior pubic ramus
Vascular
corona mortis : anastomosis of external iliac (epigastric) and internal iliac (obturator) vessels - at risk with lateral dissection over superior pubic ramus
Letournel Classification
Judet and Letournel: most common referenced classification system - classifed as 5 elementary and 5 associated fracture patterns
Most common fracture patterns
Younger : posterior wall or transverse fracture “family”
(transverse, T-type, transverse + posterior wall)
Elderly : anterior column (e.g., quadrilateral plate fractures), anterior column, posterior hemitransverse, assoicated both column fractures
Posterior wall
• Most common
• “gull sign” on obturator oblique view
Posterior column
• check for injury to superior gluteal NV bundle
Anterior wall
• Very rare
Anterior column
• More common in elderly patients with fall from standing (most common in elderly is “anterior column + medial wall”)
Transverse
• Axial CT shows anterior to posterior fx line
• Only elementary fx to involve both columns
Associated Both Column
• Characterized by dissociation of the articular surface from the inonimate bone
• “spur sign” on obturator oblique
Transverse + Post. Wall
• Most common associated fx
T Shaped
• May need combined approach
Anterior column or wall + Post. hemitransverse
• Common in elderly patients
Post. column + Post. wall
• Only associated fracture that does not involve both columns
Treatment
Nonoperative: protected weight bearing for 6-8 weeks
indications: patient factors - high operative risk (e.g., elderly patients, presence of DVT), morbid obesity, open contaminated wound, late presenting > 3 weeks
fracture characteristics: minimally displaced fracture (< 2 mm), < 20% posterior wall fractures (treatment based on size of posterior wall is controversial - recommend an exam under anesthesia (EUA) using fluoroscopy best method to test stability), Femoral head congruency with weight bearing roof (out of traction) - both column fracture pattern with secondary congruence (out of traction), displaced fracture with roof arcs > 45° in AP and Judet views or >10 mm on axial CT cuts
technique
- Skeletal traction NOT required if stable fracture pattern, outside the weight-bearing dome
- activity as tolerated with crutches/walker
Weight-bearing: lowest joint reactive forces seen with toe-touch weight bearing and passive hip abduction, greatest joint contact force seen when rising from a chair on the affected extremity
DVT prophylaxis if slow to mobilize
Close radiographic follow-up
Operative treatment
open reduction and internal fixation
indications
- Patient factors: < 3 weeks from date of injury, physiologically stable, adequate soft-tissue envelope, no local infection, pregnancy is not contraindication to surgical fixation
- Fracture factors: displacement of roof (> 2 mm), unstable fracture pattern (e.g. posterior wall fracture involving > 40-50%), marginal impaction, intra-articular loose bodies, irreducible fracture-dislocation
Approaches
- Anterior : ilioinguinal, iliofemoral, modified stoppa
- Posterior: Kocher-Langenbach
- Combined: extended ilifemoral
Fixation modalities:
- Column fixation strategies: reconstruction bridging plate and screws, percutaneous column screws, cable fixation
- Wall fixation strategies: bridge plate and screws, lag screw and neutralization plate, spring (butress) plate
outcomes
timing: associated hip dislocations should be reduced within 12 hours for improved outcomes
worse outcomes with fixation of fracture > 3 weeks from time of injury - earlier operative treatment associated with increased chance of anatomic reduction
Peri-operative : clinical outcome correlates with quality of articular reduction
Postoperative CT scan is most accurate way to determine posterior wall accuracy of reduction which has greatest correlation with clinical outcome - ideally articular reduction <2mm
Post-operative
greatest stress on acetabular repair occurs when rising from a seated position using the affected leg, and occurs in the posterior superior portion of the acetabulum
Functional outcomes most strongly correlate with hip muscle strength and restoration of gait postoperatively
Total hip arthroplasty
indications
usually elderly patients with significant osteopenia and/or significant comminution, pre-existing arthritis, post-traumatic arthritis in all ages
techniques
timing: immediate vs. delayed THA
immediate THA (with, or without, fracture fixation)
wall fractures : butress plate with multi-hole cup
column fracture: cage and cup constructs
delayed THA
outcomes: patients older than 60 years have approx. a 30% late conversion rate to THA after acetabular fractures
10-year implant survival noted to be around 75-80%
A 65-year-old diabetic male with forefoot gangrene is evaluated for possible amputation. When discussing the amputation levels with the patient, which of the following should be noted to require the greatest increase in energy expenditure for ambulation?
Syme amputation Unilateral transtibial amputation Transfemoral amputation Bilateral transtibial amputations Through the knee amputation
.A transfemoral level amputation requires the greatest increase in energy expenditure of the amputation levels given, and a Syme amputation the least.
Biologic joints are energy couples. When performing amputation surgery, more proximal amputations, accompanied by the removal of more joints, decreases the ability of patients to walk and live independently. Therefore a Syme amputation would require only a small increase in energy needed for ambulation. Bilateral transtibial amputee patients have an energy expenditure that is 40% above baseline levels. According to Miller’s Review text, unilateral transfemoral amputee’s require a 65% increase in energy expenditure.
Pinzur et al. retrospectively studied 97 adult patients with diabetes mellitus who underwent Syme ankle disarticulation because of a neuropathic foot with an infection or gangrene, or both, during an 11 year period. They found that overall 84.5% of the patients achieved wound healing. Their data support the value of Syme ankle disarticulation in diabetic patients with infection or gangrene.
Laughlin et al. retrospectively reviewed the surgical results and functional outcome of 52 patients treated with Syme amputations for forefoot gangrene. Wound healing was correlated with the preoperative status of the posterior tibial artery and follow-up averaged 27 months. The authors found that that posterior tibial artery Doppler examination is predictive of healing in the Syme amputation performed on diabetics, and that furthermore, diabetics can attain a functional level of ambulation with a Syme amputation.
Illustration A shows a clinical photo of a standard Syme amputation with a healthy heel pad which can allow for limited weight bearing in the absence of a prosthesis.
Incorrect Answers:
1,2,4,5: All of these amputation levels have lower energy expenditure increases than a transfemoral amputation.
A Syme amputation is effectively a tibiotalar disarticulation, which provides an end-bearing stump that could potentially allow ambulation without a prosthesis over short distances. It works better for tumor and trauma, but the heel pad must be viable. The two most common problems are 1) skin sloughing from compromised vascular supply and 2) migration of the heel pad due to instability. A hypermobile heel pad can cause difficulty with prosthesis wear and damage to the soft tissues which can eventually lead to failure. Both malleoli are usually removed in the procedure, except in children or during the first stage procedure of a diabetic or infection case. The tibialis anterior is usually tenodesed to the anterior heel pad along with the EDL tendon to avoid posterior migration of the heel pad.
Ankle/Foot Amputation:
Syme amputation (ankle disarticulation) - patent tibialis posterior artery is required. More energy efficient than midfoot even though it is more proximal
Stable heel pad is most important factor
Used successfully to treat forefoot gangrene in diabetics
Technique: medial and lateral malleoli are removed flush with distal tibia articular surface. The medial and lateral flares of the tibia and fibula are beveled to enhance heel pad adherence. Heel pad is secured to anterior tibia
Pirogoff amputation (hindfoot amputation): removal of the forefoot and talus followed by calcaneotibial arthrodesis
calcaneus is osteotomized and rotated 50-90 degrees to keep posterior aspect of calcaneus distal
Allows patient to mobilize independently without use of prosthetic
Chopart or Boyd amputation (hindfoot amputation)
a partial foot amputation through the talonavicular and calcaneocuboid joints. Primary complication is equinus deformity - avoid by lengthening of the Achilles tendon and transfer of the tibialis anterior to the talar neck
leads to apropulsive gait pattern because the amputation is unable to support modern dynamic elastic response prosthetic feet
Lisfranc amputation (midfoot amputation) - equinovarus deformity is common caused by unopposed pull of tibialis posterior and gastroc/soleus. Prevent by maintaining insertion of peroneus brevis and performing achilles lengthening. A walking cast is generally used for 4 week to prevent late equinus contracture. Energy cost of walking similar to that of BKA
Transmetatarsal amputation: more appealing to patients who refuse transtibial amputations
Almost all require achilles lengthening to prevent equinus
Great toe amputations: preserve 1cm at base of proximal phalanx - preserves insertion of plantar fascia, sesamoids, and flexor hallucis brevis - reduces amount of weight transfer to remaining toes so lessens risk of ulceration
Metabolic cost of walking
- increases with more proximal amputations
perform amputations at lowest possible level to preserve function
- exception: Syme amputation is more efficient than midfoot amputation
- inversely proportional to length of remaining limb
Ranking of metabolic demand (% represents amount of increase compared to baseline)
Syme - 15%
transtibial: traumatic - 25% average, short BKA - 40%, long BKA - 10%, vascular - 40%
transfemoral: traumatic - 68%, vascular - 100%
thru-knee amputation: varies based on patient habitus but is somewhere between transtibial and transfemoral
Most proximal amputation level available in children to maintain walking speeds without increased energy expenditure compared to normal children
Transfemoral Amputation:
- Maintain as much length as possible, however, ideal cut is 12 cm (10-15cm) above knee joint to allow for prosthetic fitting
Technique
- 5-10 degrees of adduction is ideal for improved prosthesis function
- adductor myodesis: improves clinical outcomes
creates dynamic muscle balance (otherwise have unopposed abductors). Provides soft tissue envelope that enhances prosthetic fitting
Gritti-Stokes amputation: amputation through the femur near level of adductor tubercle. Synovium is excised to prevent postoperative effusion. Patella is arthrodesed to the end of femur for improved end bearing
prepatellar soft tissue is maintained without iatrogenic injury - improved outcomes as compared to transfemoral amputation
Wound Healing Dependent on - vascular supply - nutritional status - immune status
Improved with
- albumin > 3.0 g/dL
- ischemic index > .5 measurement of doppler pressure at level being tested compared to brachial systolic pressure
- transcutaneous oxygen tension > 30 mm Hg (ideally 45 mm Hg)
- toe pressure > 40 mm Hg (will not heal if < 20 mm Hg)
ankle-brachial index (ABI) > 0.45
total lymphocyte count (TLC) > 1500/mm3
- Hyperbaric oxygen therapy
contraindications include: chemo or radiation therapy
pressure-sensitive implanted medical device (automatic implantable cardiac defibrillator, pacemaker, dorsal column stimulator, insulin pump) , undrained pneumothorax
Complications
- Wound healing
- Contractures: adjacent joint contractures are common
prevent with early aggressive mobilization and position changes
- Heterotopic ossification: more common in trauma-related setting
- Infection: trauma-related amputation have an infection rate of around 34%
- Postamputation Neuroma : occurs in 20-30% of amputees
prevent with proper nerve handling at the time of procedure. Treatment: targeted muscle rein nervation - a method of guiding neuronal regeneration to prevent or treat post-amputation neuroma pain and improve patient use of myoelectric prostheses
- Phantom limb pain
occurs in 53-100% of traumatic amputations
mirror therapy is a noninvasive treatment modality
- Bone overgrowth: most common complication with pediatric amputations. Treatment: prevent by performing disarticulation or using epihphyseal cap to cover medullary canal